¿Sabemos por qué hay un límite de velocidad en nuestro universo?

Esta pregunta es sobre por qué tenemos un límite de velocidad universal (la velocidad de la luz en el vacío). ¿Hay una ley más fundamental que nos diga por qué es esto?

No estoy preguntando por qué el límite de velocidad es igual a C y no otra cosa, sino por qué hay un límite en absoluto.

EDITAR: Las respuestas como "si no fuera ..." y las respuestas que explican las consecuencias de tener o no tener un límite de velocidad no están, en mi opinión, dando una respuesta específica a si hay una forma más fundamental de derivar y explicar la existencia del limite

No es universal. Es solo un límite para la ley local de la relatividad especial.
La expansión del espacio no tiene límite de velocidad. Las partículas que viajan en el espacio-tiempo tienen un límite de velocidad de un número arbitrario c. (Otros han explicado por qué c tiene que ser finito)
Todas las respuestas son fundamentalmente respuestas como "si no fuera...", porque todas nuestras teorías se basan en postulados, que son básicamente razonamientos de este tipo. Es decir, incluso los argumentos que utilizan las matemáticas para mostrar algo se basan fundamentalmente en argumentos como este.
"Por qué hay un límite de velocidad" es una respuesta filosófica y no puede ser respondida por la física. "Por qué debería haber un límite de velocidad" es una pregunta que se relaciona con nuestra interpretación filosófica de que nuestro Universo es determinista (necesitamos una forma de preservar la causalidad). La velocidad de la luz, también conocida como "C" o 1ls/s. Es un límite de velocidad impuesto axiomáticamente en la teoría de la relatividad de Einstein para preservar la causalidad. Entonces, si queremos preservar la causalidad en la teoría de la relatividad, debemos imponer un límite de velocidad universal, que garantice que los eventos sucedan en un orden causal y determinista.

Respuestas (34)

Imagina que hay una persona que prefiere medir la cantidad de dinero en su cuenta bancaria con el valor V . la ecuacion es V = C bronceado norte , dónde norte es la cantidad real de dinero en dólares. Esta persona también estará confundida:

¿Por qué hay un límite ( C ) sobre la cantidad de dinero que puedo tener? ¿Hay alguna ley que diga el valor de mi dinero, V , no puede ser más que C ?

La respuesta es que simplemente está usando una variable "incorrecta" para medir sus activos. V no es aditivo — es una transformada de una variable aditiva, norte , que tiene que usar para que todo tenga sentido. Y no hay "ley del universo" que limite el valor de V — tal límite es solo un producto de su propia terquedad.

Lo mismo se aplica para medir la velocidad: es la variable "incorrecta" para describir la velocidad del movimiento; la velocidad no es aditiva. La variable "correcta" se llama " rapidez ": es aditiva y no tiene límite.

Los comentarios no son para una discusión extensa; esta conversación se ha movido a chat .
No me gusta mucho esa respuesta. Si mido el tiempo que le toma a la luz ir a la luna y regresar, puedo medir que la distancia/tiempo es c. Así que esa es la definición de velocidad. Es una medida finita. Extraer un valor infinito de esas 2 medidas simples de duración y distancia parece artificial en el mejor de los casos.
@ njzk2 Debería dar más detalles sobre la medición de la distancia. No es tan "simple" como podrías pensar.

Hay un documento maravilloso que recuerdo haber leído que usa solo álgebra básica solo para determinar la forma más general de la fórmula para sumar velocidades, basado solo en principios generales de simetría (lo que funciona aquí también funciona allí , etc.).

No puedo encontrar ese, pero es fácil encontrar Nothing but Relativity . Y otras que se derivan de una versión inicial de Mermin.

Otro artículo con la misma idea (pero diferentes axiomas específicos) es Una derivación más de la transformación de Lorentz de Jean-Mark Lévy-Leblond publicado en 1976 (gracias bdforbs ).

Al final, muestra que la conocida relatividad especial es la respuesta ineludible. En el papel un valor arbitrario no determinado Z salió de eso. Hay 3 casos: negativo no funcionó (en el artículo que recuerdo, las matemáticas se ahogan. En el artículo vinculado, "no es coherente", pero Greg Egan resolvió este caso en detalle ). 0 da el tiempo absoluto fijo de Galileo, y cualquier valor positivo da relatividad especial con un límite de velocidad.

La velocidad es simplemente . En unidades naturales es un valor de 1 . La razón por la que la velocidad de la luz (o de cualquier cosa sin masa) parece ser algo específico es su relación con otras cosas. Al final, puede encontrar que la relación llamada constante de estructura fina tiene un valor particular.

Su verdadera pregunta es: ¿ por qué la estructura fina es constante el valor que es? La respuesta es desconocida. Puede estar determinado por un conjunto de reglas más profundo de lo que conocemos ahora, puede provenir de la física de la que no tenemos conocimiento, o puede ser un puro accidente como el número de planetas en nuestro sistema solar en lugar de una ley.


Entonces, ¿por qué es Z distinto de cero? Bueno, si es cualquier valor aleatorio, tiene una posibilidad muy pequeña de ser exactamente cero, y el valor real que tiene, siempre que no sea exactamente cero, solo establece un factor de escala y realmente no significa nada.

La riqueza del Universo se debe a efectos emergentes de los fundamentos: dado que el tiempo es relativo combinado con la mecánica cuántica significa que deben existir antipartículas , junto con la producción de pares y la aniquilación.

Si el universo no tuviera relatividad especial pero tuviera un tiempo absoluto fijo, sería muy diferente y no tendríamos el mismo tipo de cosas en absoluto. Todo encaja y, en general, si observa una función profunda, encontrará que está necesariamente basada en las otras funciones profundas. Es todo o nada: no puedes elegir la velocidad finita de la causalidad por sí misma más de lo que puedes preguntar por qué un lado de un triángulo tiene la longitud específica que tiene.


notas

El documento vinculado y otros relacionados (¿de Mermin?) Usan un valor K de una manera ligeramente diferente a la Z en el documento que no puedo encontrar ahora. Estos fáciles de encontrar también usan cálculo y límites, que para este propósito no es tan satisfactorio como usar álgebra sola con cuatro relaciones supuestas debido a la simetría.

El que recuerdo usado originalmente (IIRC)

  1. el espacio es el mismo aquí y allá
  2. si A ve que B se mueve con velocidad X, B ve que A se mueve con velocidad X en la dirección opuesta. equivalente al espacio es el mismo en todas las direcciones en un modelo unidimensional.
  3. los resultados de los experimentos ahora son los mismos que los resultados de los experimentos en diferentes momentos. (Igual que #1 pero para T en lugar de X)
  4. igual que #2 pero para T en lugar de X

Creo que esto es más satisfactorio que el artículo de Lévy-Leblond, que toma como axioma el acuerdo universal de causalidad.

Esa es una especie de enfoque antrópico, pero no es menos cierto. Parece contradictorio, pero solo porque vemos las cosas en una escala más pequeña donde las reglas parecen ser diferentes. Sería como preguntar cómo las personas en la parte inferior de la tierra pueden permanecer unidas si la tierra es redonda. Ese razonamiento proviene de lo que entendemos intuitivamente, pero eso no significa que se aplique lo mismo a mayor escala.
Comenzando con consideraciones de simetría, puede demostrar que, de hecho, solo tiene la relatividad especial como posible respuesta, pero c = infinito no se descarta en absoluto.
@Neil Es bastante cierto, pero el principio que entendemos intuitivamente en este caso es la causalidad: es un poco difícil dar sentido a un universo no causal, que es lo que surgiría si se eligiera el signo negativo. Además, este no es el único efecto, y todos los efectos extraños que mostraría un universo riemanniano se estudian en la trilogía "Ortogonal" de Greg Egan. Egan da un resumen de algo de la física aquí .
Creo que el maravilloso artículo que mencionas es Cómo Galileo pudo haber derivado la teoría especial de la relatividad .
@Ruslan no, ese no es el indicado. Las ondas manuales "usan simetría", mientras que la que recuerdo se trataba de usar 4 identidades (traducción y reflexión en el espacio y el tiempo) para obtener la función.
Mermin no originó esto. Se remonta a un artículo de 1911 de Ignatowsky.
¿Es "Una derivación más de la transformada de Lorentz"? doi.org/10.1119/1.10490
“Bueno, si se trata de cualquier valor aleatorio, tiene una probabilidad muy pequeña de ser exactamente cero”. No estoy seguro de que sea un buen argumento. Una "cantidad" que es cero (es decir, en cierto sentido "no existente") en una teoría parece hacerla más parsimoniosa y, por lo tanto, según la navaja de Occam, más probable a priori.

La mejor respuesta que se me ocurre es "porque, de lo contrario, el Universo sería fundamentalmente impredecible".

Podemos imaginar el espacio-tiempo como una variedad de cuatro dimensiones METRO ; las leyes de la física dictan entonces cómo se comportan la materia y la energía en esta variedad. (Por el bien del argumento, puede ver esto como el antiguo espacio plano de Minkowski, aunque el argumento se generaliza para incluir también espaciotiempos curvos). Entonces podemos hacer la siguiente pregunta: "Supongamos que sé cómo se comportan la materia y la energía en alguna porción finita del universo en algún momento del tiempo t = 0 . ¿Qué me dice esto sobre el comportamiento de la materia y la energía en el Universo después de ese tiempo?"

Si hay un límite de velocidad para el universo, entonces existe una región del espacio-tiempo llamada dominio de dependencia , en la que se puede predecir lo que sucederá después de nuestro momento inicial. Consiste en todos los eventos del espacio-tiempo cuyos conos de luz pasados ​​1 , cuando se remontan a t = 0 , están enteramente contenidos en la región del espacio de la que teníamos conocimiento. Vista como una función del tiempo, la región del espacio que se encuentra en el dominio de dependencia se reducirá gradualmente a nada, a medida que las influencias desde fuera de nuestra región inicial (de la que no teníamos conocimiento) se propaguen hacia el interior. Pero si el universo tiene un límite de velocidad en todos los puntos, entonces tenemos la garantía de que existe un volumen finito de espacio-tiempo en el que podemos predecir lo que sucederá.

Sin embargo, si hay una velocidad de propagación infinita del Universo, entonces el dominio de dependencia se desvanece. En términos generales, no hay forma de que podamos predecir nada, porque las influencias causales podrían propagarse desde fuera de nuestra región de datos iniciales y estropearlo todo el instante después. t = 0 . Por lo tanto, si no hubiera un límite de velocidad, entonces el Universo sería básicamente impredecible; sin el conocimiento de todo lo que estaba sucediendo en el Universo en un instante determinado, las leyes de la física no tendrían poder predictivo.

Admitiré libremente que esto no es tanto una respuesta a "¿por qué hay un límite de velocidad?" como "¿cómo sería el universo si no hubiera un límite de velocidad?" Aún así, un Universo sin límite de velocidad es lo suficientemente extraño e incomprensible como para hacerme feliz de vivir en un Universo con uno. (Esto tiene matices del argumento antrópico: tal vez en algún Universo paralelo, alguna criatura incomprensible esté discutiendo sobre lo horrible que sería vivir en un Universo con un límite de velocidad).

Finalmente, tenga en cuenta que nada en este argumento se basa en la relatividad especial; todo lo que se requiere es que haya una noción de "cono de luz" en cada punto del espacio. El límite de velocidad podría variar de un punto a otro, o diferir con la dirección, pero siempre que separe la vecindad de cada punto del espacio-tiempo en un pasado causal, un futuro causal y regiones causalmente desconectadas (como con los conos de luz convencionales en la relatividad convencional) , entonces el argumento todavía sigue.


1 "Cono de luz" aquí no significa necesariamente "la trayectoria de todos los rayos de luz trazada en el tiempo", sino más bien "la trayectoria de todos los rayos que viajan al límite de velocidad trazada en el tiempo".

Siempre que la fuerza de los efectos llegue a 0 con la distancia, no veo ninguna razón por la cual esta idea de "dominio de dependencia" sea una necesidad. Ya no podemos predecir exactamente lo que sucede en cualquier volumen finito de espacio, y estamos de acuerdo con eso, entonces, ¿por qué cambiaría algo?
@immibis: No hay razón para esperar que la fuerza de los efectos llegue a 0. De hecho, si está observando todas las cosas que podrían afectarlo dentro de "un tictac de reloj", esperaría que estén más lejos (lo que tendría que moverse más rápido para alcanzarte) para crecer cuadráticamente en fuerza vs distancia.
esta respuesta es profunda. bueno
@R .. La radiación electromagnética y la gravedad, ambos decaen como 1 r 2 - el decaimiento cuadrático cancela la cantidad cuadrática de cosas que podrían afectarte, por lo que las cosas a 500 mil millones de kilómetros de distancia no pueden afectarte más que las cosas a 5 cm de distancia. (El efecto total sigue siendo ilimitado en teoría, pero en la práctica, el espacio a 500 mil millones de kilómetros de distancia se llena en una proporción mucho menor)
En cambio cosa de una partícula.
¿No es el Universo ya fundamentalmente impredecible? Desintegración radiactiva: no tiene idea de cuándo se desintegrará exactamente un solo átomo. Si tiene uno de estos en su parte observable del espacio, no puede predecir cómo se verá en t = 1 porque puede o no haberse descompuesto. Entonces, con una posibilidad infinitamente pequeña, todo su experimento podría explotar. Lo mismo con las fluctuaciones del vacío cuántico...
Por otro lado, no veo ningún problema en la práctica, si el universo es teóricamente impredecible. En casi todas nuestras simulaciones y predicciones del mundo real, suavizamos eventos externos poco probables. Si quiero predecir dónde estará la luna mañana, NO tengo en cuenta que alguien puede explotarla mientras tanto, incluso si esto es posible a una velocidad inferior a la de la luz. Esto sería lo mismo: por supuesto, en cualquier momento podría aparecer algo extremadamente rápido y cambiarlo todo, pero necesitaría tanta energía para ser tan rápido que en realidad no sucedería en ningún momento...
@immibis A escalas razonables, el universo parece uniforme. Entonces, el caparazón del universo de un millón de años luz de espesor a 1 mil millones de años luz, 10 mil millones de años luz, 100 mil millones de años luz, 1000 mil millones de años luz, 10000 mil millones de años luz, 100000 mil millones de años luz, todos contribuyen en la misma cantidad. La primera de estas capas genera un solo fotón que golpea tu cuerpo durante tu vida. Bueno, hay un número infinito de tales conchas. Y algo multiplicado por el infinito va a arruinar tu día.
Perspectiva interesante. Eso parece sugerir que la única razón por la que el tiempo no es absoluto es porque terminarías con muchos objetos arbitrarios al comienzo del universo que no son la causa de nada más, cuando en realidad cada efecto puede ser el resultado de una sola inicial. "causa". Cuando un estanque está lleno de olas y ruido, tiene sentido suponer que comenzó con una sola roca cayendo, no que siempre haya sido así.
¿Qué quieres decir con "predecir"? no podemos predecir el futuro en la vida real, por lo que me causa confusión
La incomprensible criatura señala que el conocimiento de todo lo que sucedía en el Universo en un determinado instante de tiempo es mucho más fácil cuando no existe un límite universal de velocidad.

La física es una disciplina científica en la que las observaciones y las mediciones se ajustan a modelos matemáticos que describen los datos existentes y predicen con éxito nuevos valores para nuevas condiciones de contorno. Cuando esto sucede se dice que el modelo ha sido validado.

Si nuevos experimentos y observaciones falsearan el modelo, habrá que volver a examinar las suposiciones e incluso buscar un nuevo modelo.

En la actualidad, el modelo validado que tenemos para las partículas elementales es el Modelo Estándar , que utiliza la mecánica cuántica relativista y ha sido probado innumerables veces con experimentos de laboratorio y de observación. Este modelo matemático, debido a que incorpora la relatividad especial, concuerda con la observación de que la velocidad de la luz es una constante c en el vacío. Es cierto que el valor de c es fortuito para esta discusión. Lo que se cuestiona es la existencia del límite, y la única respuesta posible es: porque el modelo teórico concuerda con el experimento y es muy predictivo.

Si nuevos datos desvirtúan el modelo estándar hasta el punto de ser necesario un nuevo modelo teórico, este nuevo modelo deberá incorporar la estructura existente para los casos que haya sido validado, incluyendo el límite de velocidad de la luz. El modelo estándar se convertiría en un caso límite para la nueva teoría, para las energías y condiciones de contorno que fueron validadas, de manera similar a como la física newtoniana emerge de la relatividad especial en el límite de bajas energías.

@annav Entonces, la conclusión es que la explicación más fundamental que tenemos ahora mismo es que "simplemente es", ¿verdad?
@LandosAdam Sí. La teoría que exige una velocidad limitada y fija para partículas de masa cero se deriva de los datos. La física no responde en última instancia a las preguntas de "por qué". Dice "cómo" con el modelo aceptado se puede llegar a una observación. Si hubiéramos observado una velocidad variable de la luz, habríamos desarrollado una teoría diferente.
@annav lo sé, esa es la belleza de su objetividad en cierto sentido. Pero, a veces, una teoría puede explicar algunas cosas que antes se consideraban fundamentales. Por eso hice esta pregunta. Y, tal como lo veo, el hecho de que exista la posibilidad de que pueda derivarse de leyes más fundamentales de las que aún no somos conscientes me entusiasma más. Pero, la palabra clave aquí es "PODRÍA", porque también podría ser como usted dijo.
@LandosAdam: exactamente. Si preguntamos, "¿por qué los objetos se mueven alrededor/más allá del Sol en secciones cónicas?", entonces la física puede decir "por qué", es decir, en términos de otras propiedades del modelo que pueden verse en cierto sentido como más fundamentales: una inversa -La aceleración cuadrada da como resultado ciertas características geométricas. Por supuesto, todavía no es una respuesta "definitiva" a "por qué", pero es una respuesta. Pero cuando se trata de por qué la luz no llega instantáneamente, anna afirma aquí que solo hay una respuesta posible, es decir, el modelo no proporciona ninguna base "más profunda" para el fenómeno.
También vale la pena tener en cuenta que si algo responde o no a una pregunta de "por qué" es algo subjetivo, ya que estamos hablando de simplificar las explicaciones ("los múltiplos de 10 terminan en 0 porque los estamos escribiendo en base 10"), no causas físicas ("Me duele el pie porque es más blando que la roca que acabo de patear"). Así que otros han concluido que hay una respuesta al "por qué", y de hecho pueden hablar de características del modelo que para ellos explican parcialmente la existencia en el modelo de un límite.
@SteveJessop estoy de acuerdo con lo que dices, aunque no veo por qué la respuesta a "cuál es la razón más fundamental para que suceda esta cosa X" es subjetiva. No es una cuestión de opinión. Parece una cuestión de opinión porque algunas personas saben más que otras, por lo que obtenemos muchas respuestas. Pero estoy seguro de que si profundiza en la mayoría de las respuestas, encontrará la respuesta más fundamental en esta página, y en este caso es que no sabemos el por qué detrás de esto (si es que hay un por qué).
@LandosAdam: Si no ve por qué la respuesta a "cuál es la razón más fundamental de X" es subjetiva, entonces debe pensar cuál es su definición de "fundamental" . Te garantizo que no es objetivo.
@QuanticMan acaba de ver esta discusión. La física usa las matemáticas. En matemáticas, debe saber que la elección de los axiomas es subjetiva y, por lo general, depende de minimizar los pasos. Es cierto para una teoría matemática axiomática que se podría sustituir un axioma por un teorema, y ​​entonces un axioma se convierte en un teorema, es decir, demostrable a partir de los axiomas. En física hay postulados que eligen el subconjunto de las matemáticas axiomáticas que se ajusta a los datos. En el mismo sentido que con los axiomas uno podría tomar diferentes postulados y usar las matemáticas para demostrar los postulados anteriores, una parte subjetiva.
@annav, tus comentarios y publicaciones SIEMPRE van directo al punto y son muy perspicaces. Sigan publicando... REALMENTE disfruto leyendo sus respuestas.
He tenido esta teoría durante mucho tiempo. Creo que la velocidad de la luz está limitada a lo que es porque el flujo del TIEMPO es exactamente el mismo (en otras unidades) y cualquier cosa más rápida que la luz atravesaría el TIEMPO hacia el futuro (o hacia ninguna parte) para nunca más ser visto. Incluso cuando el futuro se convierte en el AHORA, la COSA todavía se está moviendo hacia el nuevo futuro del nuevo AHORA. Entonces, si toda la luz se moviera más rápido, NO se vería ninguna luz y no estaríamos aquí.
@annav ¿Quiere decir que, incluso si logramos la comprensión más fundamental de nuestro Universo que podamos, alguien podría responder a esta pregunta cambiando un axioma con un teorema y dando una explicación diferente? Es por eso que una explicación de un por qué "fundamental" se considera como algo subjetivo. (Además, ¿podríamos extender esta conversación en una sala de chat?)
Las ideas teóricas no existen en un vacío teórico. Las apelaciones a la experimentación no son las únicas respuestas posibles a las preguntas de "por qué".
@BenCrowell Evidentemente no estamos de acuerdo. Las preguntas de "por qué" después de pasar por "cómo se ajustan matemáticamente al modelo teórico" terminan en los postulados/principios/leyes que son la destilación de observaciones/experimentos, axiomas adicionales impuestos a las matemáticas para recoger el subconjunto de soluciones relevantes. a los datos Entonces la única respuesta es "porque eso es lo que se ha observado".
@annav ¿Por qué explicar algo en absoluto? ¿Por qué formular cualquier teoría en absoluto? ¿Solo para que pueda recordar fácilmente los datos? Supongo que ahora tenemos buenas computadoras, así que no tenemos que recordar cosas. Entonces, creo que una gran tabla de todas las observaciones es la mejor Física que podríamos hacer en su universo. Estoy feliz de no vivir en tu universo.
La Física Teórica no es una mnemotécnica para recordar los resultados experimentales. En cualquier momento, la teoría más básica que tenemos es precisamente la respuesta a por qué suceden todos los fenómenos que pueden ser explicados por esta teoría básica. Sería estúpido o deshonesto decir, "no podemos explicar por qué tenemos una ley de fuerza del cuadrado inverso para la gravedad o el electromagnetismo más allá de la afirmación de que los encontramos así". Los principios básicos de la teoría GR y YM son la razón por la que tenemos estas leyes del cuadrado inverso.
@Dvij No estoy de acuerdo. Primero estudiamos y encontramos el comportamiento 1/r^2 y luego lo usamos para PREDECIR el comportamiento futuro. Los bancos de datos, como los mapas, no tienen capacidad predictiva. Es la posibilidad de predicciones utilizando herramientas matemáticas (modelos matemáticos) lo que es importante en física. Las predicciones correctas validan la herramienta, las incorrectas provocan cambios en los modelos. Las teorías de la física se formulan de modo que se puedan predecir configuraciones futuras y la teoría se compare con nuevos datos.

Maxwell hizo un riguroso estudio matemático de las propiedades de la electricidad y el magnetismo, y demostró que debe haber un fenómeno al que llamó ondas electromagnéticas . Según la teoría de Maxwell, una onda electromagnética debe propagarse a una velocidad constante que él llamó C , y que podía calcularse a partir de otras constantes físicas conocidas y medibles en ese momento.

Alrededor de ese tiempo, los científicos estaban debatiendo activamente la naturaleza de la luz. Después de que Hertz demostrara la existencia de ondas electromagnéticas en un experimento de laboratorio, la teoría de Maxwell se convirtió repentinamente en la explicación preferida para la luz.

Lo divertido de la teoría de Maxwell era que la velocidad era relativa a quien la midió. Si tú y yo medimos la velocidad de las ondas que emanan de la misma fuente, ambos deberíamos obtener el mismo resultado, independientemente de nuestro movimiento en relación con la fuente o entre nosotros.

Algunos físicos encontraron que eso era preocupante y probaron diferentes formas de justificarlo. El gran logro de Einstein fue combinar sus diversas ideas en una sola teoría matemática consistente y rigurosa. Una de las consecuencias de su teoría---probada en las matemáticas---es que si algo tiene una velocidad característica que debe ser la misma para todos los observadores, entonces nadie podrá observar nada que se mueva más rápido que esa velocidad.

Entonces, el límite de velocidad universal es una consecuencia matemática de ciertas constantes físicas medibles y, hasta donde se sabe, fundamentales; por ejemplo, la permeabilidad del espacio libre .

Su pregunta entonces se reduce a, "¿por qué el universo tiene esas propiedades?"

Cada vez que los físicos responden a una pregunta de "por qué", la respuesta siempre se basa en niveles más profundos de "¿por qué?"

En realidad, las ecuaciones muestran la velocidad en función de las constantes magnéticas y eléctricas. Es una observación adicional que estos no se comportan como un menium normal en relación con el cual el observador se mueve, sino que leen obstinadamente el mismo valor sin importar su movimiento.
"Lo curioso de la teoría de Maxwell era que la velocidad era relativa a quien la midió". ¿No debería ser: "no fue relativo a quien lo midió" , es decir, independiente del marco de referencia?
Lo divertido de la teoría de Maxwell era que la velocidad era relativa a quien la midió. - No creo que eso sea cierto, porque si lo fuera, el resultado del experimento de Michelson-Morley no habría sido tan impactante. ¿Me estoy perdiendo de algo?
la velocidad era relativa : Quiere decir que la velocidad calculada, c, se toma como el mismo valor para cualquier observador. Es la misma ambigüedad de oración que le dio su nombre a la relatividad , ya que Einstein señala el mismo problema en inglés. La velocidad de la onda es c con respecto a Alice, y la velocidad es c con respecto a Bob.
Esto apunta exactamente en la dirección correcta, y merece ser mucho más votado que la respuesta "superior" actual. Me encantaría que ampliaras esto para incluir los puntos básicos de las derivaciones (tanto de Maxwell como de Einstein).
@Floris, aquí solo soy un turista. Me encantaría aprender algo de física algún día. Pasé más tiempo del que me gusta admitir trabajando en "El camino a la realidad" de Roger Penrose. Mi mayor conclusión fue que la física es un tema muy amplio. Básicamente, ese libro consta de mil cien páginas de breves introducciones a algunas de las técnicas y formas matemáticas que uno tendría que aprender para comprender la física. Tal vez cuando (¡si!) me jubile...

En física, no se puede preguntar/responder por qué sin ambigüedad. Ahora, observamos que la velocidad de la luz es finita y que parece ser la velocidad más alta para la energía.

Se han construido teorías efectivas en torno a esta limitación y son consistentes ya que dependen de dispositivos de medición que se basan en tecnología/ciencias que tienen todas c incorporadas. En las ciencias modernas, a uno no le importa lo que está sucediendo, sino lo que sucede. los dispositivos miden.

La validación de estas teorías permite decir fácilmente que existe una velocidad máxima universal. De hecho, existe una velocidad máxima para un objeto en movimiento hecho de energía cuando se mide en un espacio estático. No es exacto en un universo en expansión o en otros contextos relativistas críticos. No es exacto si no es energía, es decir, con la supuesta influencia que sería intercambiada por partículas entrelazadas. Aun así, uno no puede viajar más rápido que la luz a menos que sea en sueños.

La física moderna es nueva. Se extiende en todas direcciones. Todavía es difícil hacer una síntesis de todo lo que se sabe, lo que es útil y lo que no es consistente y relevante. Quizá el valor finito de c (y no sólo de c) venga del análisis cuántico de campos profundos, del mismo modo que la relatividad siguió la riquísima teoría de Maxwell. Mientras tanto, la comunidad científica no está al tanto de un nuevo análisis de este tipo, incluso si ya se ha publicado en algún repositorio oscuro.

Eso es "¿Por qué sabemos que hay un límite de velocidad en nuestro universo?" que es una pregunta muy diferente a "¿Sabemos por qué hay un límite de velocidad en nuestro universo?"
@immibis Traté de mostrar que no hay respuesta a este por qué porque es un hecho experimental que se asume como un postulado, incluso si se puede discutir el concepto de velocidad. Quizás su estado evolucionará con más conocimiento y más síntesis, pero hoy, ¿quién sabe deducir el límite de velocidad de otros supuestos?
Hay un problema con preguntar 'por qué' en las ciencias. En el inglés cotidiano, 'por qué' y 'cómo' son a menudo sinónimos: la respuesta a "¿Por qué es azul el cielo" es la misma que a "¿Cómo es azul el cielo?". Pero para preguntas más profundas, el 'por qué' deja de tener sentido, porque es una pregunta teleológica (no teológica): en última instancia, está preguntando sobre el propósito o la meta final . Pero las leyes de la naturaleza no están diseñadas para un objetivo particular; simplemente son Por lo tanto, 'cómo' es realmente la única pregunta que la ciencia puede responder. En este caso, el cómo resulta de las ecuaciones.
@ user151841 sí, porque por qué debe responderse con una demostración lógica en la teoría actual. Pero no podemos demostrar un postulado. Este se basa en observaciones robustas y teorías experimentadas.
@ user151841 Aunque en parte estoy de acuerdo con usted y tiendo a colocar esas preguntas en alguna parte de la metafísica, no estoy de acuerdo con que lo reduzca a un argumento teleológico. No es lo mismo preguntar por qué el valor de π es lo que es, lo cual no tendría sentido. Más bien, puede darse el caso aquí de que la alternativa ni siquiera sea consistente, en un sentido matemático, lo que proporcionaría una respuesta que no se basa en argumentos teleológicos.
@G.Bergeron No estoy en desacuerdo; para audiencias sofisticadas, las explicaciones que son epifenominales de fenómenos más simples, o que son comportamientos o propiedades emergentes, suelen ser satisfactorias para una pregunta de "por qué". Sospecho que esto es lo que "realmente" están preguntando
Si la física no nos da una respuesta a la pregunta "¿por qué" deberíamos leer los libros religiosos para aprender la respuesta?

Esta pregunta ha generado algunas respuestas interesantes, y también me gustaría hacer una contribución. Debería quedar perfectamente claro que vivimos en un mundo con una velocidad máxima finita, y muchas respuestas se han referido a las consecuencias y razones de esto.

Sin embargo, me gustaría señalar un aspecto que parece haberse olvidado por completo en las otras respuestas. Si la velocidad de la luz fuera infinita, no tendríamos luz en absoluto.

Para ver esto, eche un vistazo a las ecuaciones de Maxwell nuevamente. Nótese que en ellos C = 1 m 0 ϵ 0 , por lo que si establece C entonces cualquiera (o ambos) de m 0 y ϵ 0 tendria que ser cero. Esto matará efectivamente la existencia de campos magnéticos dinámicos.

Especialmente, para la luz, significa que × B = 1 C 2 mi t 0 , por lo que los campos magnéticos serían estáticos (y de intensidad cero, recuerde que no hay monopolos magnéticos). Por lo tanto, lo único que quedaría del electromagnetismo sería simplemente electrostática.

Físicamente esto también tiene sentido, si C entonces la respuesta del campo eléctrico a cualquier reordenamiento de las cargas sería instantánea, por lo que no hay lugar (¿tiempo?) para una respuesta del campo magnético.

Además, pensando en los campos magnéticos (y especialmente en la fuerza de Lorentz), también tiene sentido que los campos magnéticos desaparezcan. Si C no hay contracción de longitud, por lo que no habrá fuerzas de Lorentz en ninguna partícula.

Por lo tanto, cuando hablamos de señales que se propagan infinitamente rápido, es dudoso a qué señales nos referimos.

Creo que su respuesta es la primera, que realmente muestra lo que se descompone cuando C tiende al infinito, sin invocar realmente el argumento circular de "causalidad porque SR, y con infinito c no hay causalidad", etc. Gracias y +1
Para ser honesto, realmente no me convence el argumento de que C significa que no hay causalidad. Este es solo el límite newtoniano, donde el tiempo es absoluto, y esto funciona bien hasta donde yo sé.
El infinito es mucho mayor que la velocidad de la luz. ¿Por qué la velocidad de la luz es la misma en todas partes y todo el tiempo? Porque el espacio cambia de densidad, lo cual se observa mediante el corrimiento al rojo y cosas por el estilo, pero ¿no cambia también el tiempo (la velocidad de la luz)?
@CeesTimmerman: Diría que no estamos seguros de por qué C tiene el mismo valor en todo el espacio-tiempo. El argumento más fuerte sería el del principio general de la relatividad. Sin embargo, estoy seguro de que si la velocidad de la luz dependiera de la posición (del espacio-tiempo), obtendría algunos efectos extraños, que con suerte serían detectables desde la Tierra.
Decir que "la velocidad de la luz no puede ser infinita" no es lo mismo que decir "nada puede ir más rápido".
@Floris: estoy de acuerdo, esto último es una consecuencia de la relatividad especial (o general). Así que mientras creamos en eso, nada puede ir más rápido que la velocidad de la luz, ni siquiera los neutrinos ;). La primera afirmación es más un hecho experimental. La velocidad de la luz es finita. Sin embargo, esto no impide que haya teorías autoconsistentes en el límite donde C . Esta publicación solo muestra uno (de muchos) ejemplos de por qué este límite es mucho más aburrido que el de finito C .

¿Por qué tenemos un límite de velocidad universal? ¿Hay una ley más fundamental que nos diga por qué es esto?

Las leyes más fundamentales son la causalidad y la localidad. La causalidad expresa el hecho (o suposición) de que los efectos no pueden preceder a las causas, y la localidad expresa el hecho (o suposición) de que las relaciones causales fundamentales se describen mediante ecuaciones diferenciales.

Dados estos dos principios fundamentales, la lógica de las matemáticas dicta que las ecuaciones diferenciales son parabólicas (como la ecuación de calor) o hiperbólicas simétricas (como la ecuación de onda).

Si son parabólicos, no hay límite de velocidad. Por ejemplo, de acuerdo con la ecuación del calor, el calor se propaga instantáneamente a lugares arbitrariamente lejanos, aunque se suprime exponencialmente con la distancia.

Si son hiperbólicos simétricos, la teoría matemática implica una velocidad de propagación finita. Por ejemplo, este es el caso de las ecuaciones de Maxwell, que limitan la velocidad de las señales electromagnéticas a un número llamado velocidad de la luz.

Es un hecho experimental que la Naturaleza se comporta de acuerdo con la segunda posibilidad, incluso independientemente de las consideraciones de la velocidad de la luz. Hay pruebas abrumadoras de que todos los procesos fundamentales de la naturaleza son del tipo hiperbólico simétrico. Incluso el calor: la ecuación del calor es solo la aproximación más simple, en la que se pierde el límite de velocidad. Pero las derivaciones más sofisticadas de la mecánica estadística de no equilibrio producen ecuaciones hiperbólicas simétricas, que se vuelven parabólicas solo con una mayor aproximación.

Que la velocidad límite sea la velocidad de la luz es muy probable, pero no necesariamente el caso. Está vinculado a la suposición de que los fotones no tienen masa. Si los fotones fueran masivos pero los gravitones no tuvieran masa, la velocidad de la luz sería menor que el límite teórico de las velocidades de las señales en el universo, que sería entonces la velocidad de la gravedad.

Sin embargo, según la revisión de partículas del Particle Data Group, los límites superiores de la masa de un fotón son extremadamente pequeños, y las observaciones actualmente están totalmente de acuerdo con la suposición de fotones sin masa.

¡Gracias! Estaba buscando esta respuesta. Quería preguntar si la causalidad y la localidad (hiperbolicidad) son diferentes de alguna manera. Si la hiperbolicidad proporciona un límite de velocidad superior, digamos la velocidad de la luz, también implica la prohibición de la transferencia de información superlumínica y, por lo tanto, implica causalidad. Entonces, son lo mismo, ¿no?
@NanashiNoGombe: Localidad significa describible por una ecuación diferencial. Una ecuación parabólica es local pero no causal.
Gracias por tu pronta respuesta. Estaba preguntando si la hiperbolicidad es lo mismo que la causalidad relativista. Creo que es. Por favor, corríjame si estoy equivocado.
@NanashiNoGombe: Es lo mismo en la física relativista clásica. En física cuántica, la relación es más complicada.
Gracias por esta respuesta. De lejos, el mejor.
Hola, ¿podría explicar o dar algunas referencias (artículos, libros, etc.) sobre cómo los dos principios fundamentales dictan que las ecuaciones diferenciales son parabólicas o hiperbólicas simétricas? ¿Y qué ecuaciones diferenciales?
@ApoorvPotnis: Las ecuaciones diferenciales parciales que gobiernan los campos clásicos deben tener un problema de valor inicial bien planteado. La consideración de pequeñas perturbaciones con wavelets locales altamente oscilatorias restringe la forma de los conjuntos característicos de la PDE. probablemente esté en algún lugar del tratado de 4 volúmenes de Hörmander.
Gracias Arnold por proporcionar la respuesta más esclarecedora hasta ahora. La velocidad de la luz no tiene que ver con otra cosa que con la causalidad. Es la velocidad a la que todo está sucediendo. Y su constante e invariante e igual a 1ls/s. ¡Y es así, para que se conserve la causalidad! La mejor respuesta hasta ahora, gracias señor.

La existencia del límite de velocidad está relacionada con la existencia del tiempo [ACTUALIZACIÓN: el tiempo es una medida que solo está disponible cuando C está limitado. Si no está de acuerdo, proporcione una manera de medir el tiempo cuando C es infinito antes de votar hacia abajo]. Si no hubiera límite de velocidad, todo sucedería instantáneamente. Además, cualquier ola en cualquier materia no se vería afectada y se propagaría momentáneamente. El tiempo desaparecería (así como la distancia y, en consecuencia, el espacio, por cierto).

Entonces, es lo mismo que "¿por qué hay tiempo?". La transferencia de energía instantánea, que actualmente es limitada, cambiaría el mundo tal como lo conocemos y dejaría de ser este mundo el que conocemos. La física newtoniana desaparecería como concepto, ya que la materia misma ya no funcionaría así. Así como el concepto de forma. Las consecuencias caerían sobre todo. Sin embargo, no estamos observando esto, observamos el límite.

Hay alguna separación inherente presente en la materia Universal, que le permite existir de la forma en que la conocemos/percibimos. Si existe un mundo sin límite, no emergimos en él, aparecimos aquí.

No es exactamente una respuesta, pero no hay nada más que decir.

ACTUALIZAR

En respuesta al comentario de @Davors:

Es difícil imaginar qué sucedería exactamente porque no podemos estar seguros de cuál es la estructura subyacente real de la realidad que compensa la velocidad de la luz y cómo se entrelaza con el resto de las cosas. Es decir, ¿cómo formarían la materia las otras 3 fuerzas si EM fuera instantáneo? Pero exploremos algunas opciones que apoyan la noción:

  1. Hay 4 fuerzas, y si la fuerza EM transfiriera toda la energía instantáneamente, incluso si las otras 3 fuerzas aún se mantuvieran, invalidaría la mayoría de las estructuras más grandes que los átomos de la materia que vemos ahora. Dado que en el nivel macro solo importan la gravedad y la EM, y la gravedad no tendrá mucho sentido en este escenario, entonces todos los procesos que se llevan a cabo a través de las fuerzas EM serán instantáneos.

  2. No podría existir ninguna estructura macro, y con todos los procesos EM yendo infinitamente rápidos, no habría posibilidad de saber nada sobre los estados de los electrones en los átomos. Se convertirán infinitamente en todos los estados posibles. Toda posible absorción y emisión ocurrirá a la vez. No estoy seguro incluso si los átomos resistirían.

  3. Intenta poner infinito en lugar de C en todas las relaciones y ver qué pasa. Además, como todas las velocidades pueden medirse efectivamente como una fracción de C , Entonces sí C = , todas las demás velocidades también serán infinitas sin importar el coeficiente de fracción.

  4. Vea la respuesta de @Nikos M.

Los comentarios no son para una discusión extensa; esta conversación se ha movido a chat .
¿Estás diciendo que si la causalidad no tuviera límite de velocidad, entonces todos los procesos ocurrirían a una velocidad infinita?


Dibujé la imagen de arriba para ampliar la maravillosa respuesta de Kostya.

Básicamente, imagine personas que miden la altura de los edificios en grados de ángulo de visibilidad de los edificios desde cierta distancia fija. Esto no es del todo irrazonable si fija la distancia C lo suficientemente grande en comparación con las alturas de los edificios. Sin embargo, para edificios más altos, notará que su altura angular no es aditiva. También la altura angular máxima posible se fija en un valor absoluto de 90 grados.

Esto es muy similar a la forma en que los humanos miden la velocidad: elegimos una cierta medida de "distancia/tiempo" que tiene sentido para velocidades más pequeñas, pero para velocidades más altas no es aditivo. Además, existe la velocidad "máxima" inalcanzable, la velocidad de la luz.

Sin embargo, el problema anterior se debe únicamente a la elección incorrecta de la velocidad de medición. La elección "correcta" para medir la velocidad es la "rapidez", como explica Kostya. Y la rapidez es tanto aditiva como ilimitada.

Este es probablemente más un comentario para Kostya, pero esa respuesta ya está llena. Me parece que ambas respuestas son un poco circulares. Si la no aditividad de la velocidad implica una velocidad máxima, entonces la pregunta es "¿por qué la velocidad no es aditiva?". Por lo que he visto, todos los intentos de responder a esta pregunta se basan en la limitación de la velocidad de la causalidad. Esto hace que el argumento sea circular.

¿Sabemos POR QUÉ hay un límite de velocidad en nuestro universo?

Tu pregunta es similar a:

"¿Sabemos POR QUÉ hay un límite de longitud?"

De la misma manera que necesitamos longitudes finitas para medir el tamaño o el intervalo entre dos puntos en el espacio euclidiano 3D , necesitamos una velocidad finita de la luz para medir el intervalo entre eventos en el espacio Minkowski 4D . Minkowski desarrolló su teoría para expandir la ecuación de Maxwell en cuatro dimensiones. Para poder s 2 en el espacio de Minkowski, (2), para preservar su invariancia, como una extensión del Teorema de Pitágoras, (1), que en tres dimensiones es:

(1) s 2 = X 2 + y 2 + z 2 ,

y en cuatro dimensiones se convierte en:

(2) s 2 = X 2 + y 2 + z 2 ( C t ) 2 ,
C debe ser no solo finito sino igual para todos los marcos de referencia, lo que está respaldado por la evidencia observada de que la velocidad de la luz (ondas EM) era independiente del marco de referencia de los observadores.

La velocidad de la luz necesita tener un límite, es decir, ser finita para que la Relatividad Especial funcione:

Si te remontas a los famosos experimentos mentales de A. Einstein, específicamente aquel en el que hay dos observadores, uno que no se mueve, A , de pie en una estación de tren y otra, B , moviéndose de pie en un tren, que pasa por la estación de tren.

ingrese la descripción de la imagen aquí

Ahora, cuando pasa el tren y A y B están uno frente al otro, los rayos caen en ambos lados de A , a la misma distancia. A los ve simultáneamente:

ingrese la descripción de la imagen aquí

Sin embargo, porque B se mueve con respecto a ellos, es decir, alejándose de uno y hacia el otro, los ve sucesivamente:

ingrese la descripción de la imagen aquí

...¿bien?

Bueno, NO, esto habría dado como resultado que la luz se midiera de manera diferente en diferentes marcos de referencia, algo que fue refutado por el experimento de Michelson-Morley que usó la Tierra como tren :
,

y midió la velocidad de la luz en dos direcciones perpendiculares:

ingrese la descripción de la imagen aquí

suponiendo que la luz que se mueve en la dirección que coincide con la dirección de movimiento de la Tierra tendría que ser más pequeña (similar a la persona, B , que estaba en el tren) que el otro, suposición que se demostró científicamente que era incorrecta.

En consecuencia, la velocidad de la luz es constante y cualquiera que la mida encontrará el mismo valor, independientemente de su velocidad o, en otras palabras, la velocidad de la luz es invariable . La invariancia podría contrastarse con la relatividad, por ejemplo, la relatividad del tiempo, que dicho sea de paso, se usó para describir por qué las dos personas, A y B , observe la misma velocidad de la luz, es decir, porque el tiempo de la persona en movimiento B está marcando más lento, en general, cuanto mayor sea la velocidad relativa entre los dos observadores, mayor será la diferencia de la tasa de tictac de sus relojes, es decir, la dilatación del tiempo .

Finalmente, la dilatación del tiempo podría observarse en presencia de un objeto con masa que genera un campo gravitatorio, o en términos de la Relatividad General en un espacio-tiempo alargado, que hará que el tiempo del observador ubicado más cerca del objeto de masa haga tictac en un ritmo más lento, es decir, la dilatación del tiempo y, respectivamente, el observador ubicado a una distancia mayor observará que su reloj marca un ritmo más rápido.

Como ves el tiempo es relativo, el espacio se alarga y la velocidad de la luz es la constante, con un valor finito que los "mantiene unidos" y los "sincroniza" , definiendo la simultaneidad de eventos. Además, con su ayuda, podemos definir un intervalo invariante entre dos puntos en el espacio-tiempo, es decir, entre dos eventos . Los intervalos de espacio-tiempo dependen de las separaciones temporales y espaciales de los dos puntos y pueden ser: similares al tiempo , similares a la luz (distancia temporal = distancia espacial) o similares al espacio (distancia temporal <distancia espacial) . Gracias a esa Relatividad Especiales una teoría exitosa, con una larga lista de evidencia experimental de apoyo.

Editar:

Como respuesta al primer comentario, que argumenta que la dilatación del tiempo es el resultado de la constancia de la velocidad de la luz, les presentaré un ejemplo de lo contrario, es decir, la constancia de la velocidad de la luz se puede mostrar como una consecuencia directa de la dilatación de la velocidad del tiempo:

Consideremos un reloj hipotético llamado reloj de fotones. En él, la luz se refleja de un lado a otro entre sus espejos y cada vez que la luz incide en un espejo determinado, el reloj marca una vez. Si este reloj está en movimiento inercial con respecto a un observador, entonces la dilatación del tiempo de la velocidad hará que, como todos los demás tipos de relojes, marque más lento. Sin embargo, debido a que el reloj se está moviendo, el pulso de luz trazará un camino angulado más largo entre los espejos. El resultado neto de la dilatación del tiempo de la velocidad y el aumento de la longitud del camino es que la velocidad de la luz en el reloj de fotones en movimiento permanece igual a la velocidad de la luz en el reloj de fotones en reposo. En otras palabras, la velocidad de la luz permanece constante.

Además, la transformación de Lorentz (LT), que fue derivada por Joseph Larmor [1] en 1897, y Lorentz (1899, 1904) [2], predijo directamente la dilatación del tiempo. De hecho, la dilatación del tiempo por el factor de Lorentz fue predicha correctamente por Joseph Larmor (1897)[3] mucho antes de que Einstein publicara su artículo en 1905.


Su pregunta tiende a ser un poco filosófica, por lo que sabemos el valor de C podría estar relacionado con una propiedad del propio espacio-tiempo, determinada junto con las otras Constantes Físicas Fundamentales durante el Big Bang, que es difícil de observar y darse cuenta de manera similar a por qué los peces no son conscientes de toda el agua que los rodea.

[1] Larmor, J. (1897), “Sobre una teoría dinámica del medio eléctrico y luminífero”, Philosophical Transactions of the Royal Society 190: 205-300.
[2] Lorentz, Hendrik Antoon (1899), “Teoría simplificada de los fenómenos eléctricos y ópticos en sistemas en movimiento”, Proc. Academia Ciencia Ámsterdam I: 427-443; y Lorentz, Hendrik Antoon (1904), “Fenómenos electromagnéticos en un sistema que se mueve con cualquier velocidad menor que la de la luz”, Proc. Academia Ciencia Ámsterdam IV: 669-678.
[3] Larmor, J. (1897), “Sobre una teoría dinámica del medio eléctrico y luminífero, Parte 3, Relaciones con los medios materiales”, Phil. Trans Roy. Soc. 190: 205-300

Creo que muchas de las respuestas aquí se centran en la mitad equivocada del problema. Te están diciendo cómo sabemos que hay un límite, en lugar de explicar por qué tiene que ser así.

En su mayor parte, no hay nada que impida la creación de un universo con una velocidad infinita de la luz que, por lo demás, sea similar al nuestro 1 . Sin embargo, hay una propiedad importante que debería tener dicho universo: tiene que ser finito y/o no homogéneo.

Esto está relacionado con la paradoja de Olber . Básicamente, es así:

  1. La luz de una estrella a una distancia determinada es inversamente proporcional al cuadrado de la distancia. L 1 D 2 .
  2. El número de estrellas a una distancia dada es directamente proporcional al cuadrado de la distancia. norte D 2 .
  3. La luz total de las estrellas a una distancia determinada es igual a la luz por estrella multiplicada por el número de estrellas. T luz = L norte luz = L luz estrella norte estrellas . ("luz" y "estrella(s)" son unidades aquí.)
  4. Por lo tanto, la luz total desde una distancia es la misma que la luz desde cualquier otra distancia. T L norte D 2 1 D 2 1 .
  5. Si las estrellas están homogéneamente distribuidas a través del universo, y el universo es infinito, podemos separar las estrellas en un número infinito de capas, cada capa con un brillo finito y constante. La suma de toda esta luz es infinita. D = 0 T = .

En el universo real, esto no es un problema debido a dos efectos: primero, la expansión del universo significa que todas las estrellas del universo se alejan de nosotros (en promedio), y la tasa de recesión es directamente proporcional a la distancia de la estrella. ; y segundo, porque la velocidad finita de la luz hace que la luz de estrellas lejanas tarde más de lo debido en llegar a nosotros debido a dicha expansión.

Combinado, esto significa que la luz que nos llega por segundo desde cualquier capa de estrellas dada desciende linealmente con la distancia. L 1 D . Además, esto significa que hay una distancia finita en la que todos los objetos a esa distancia se alejan de nosotros más rápido que la velocidad de la luz, por lo que nunca veremos la luz más allá de esa distancia (lo que hace que el universo sea finito). D = 0 norte , norte < T D < .

Pero si la luz viajara a una velocidad infinita, la expansión no ayudaría. Significaría que el brillo del universo está disminuyendo lentamente (las estrellas están más dispersas, por lo que el brillo por capa es menor), pero una disminución en el brillo infinito no ayuda mucho. Entonces, el universo tendría que ser finito en tamaño y/o las estrellas tendrían que ser menos densas cuanto más te alejes del centro.

Alternativamente, podríamos postular algún universo donde la luz se atenúa más rápidamente. L 1 D 3 o algo. Pero eso ya no tiene sentido geométrico y requiere un toque adicional para funcionar. Tal vez la luz sea absorbida de alguna manera por el mecanismo de expansión, y la luz sea absorbida más fácilmente ya que su potencia es cercana a cero. El espacio infinito significa que la absorción de luz proporcionalmente infinita da como resultado una tasa de expansión finita y aumenta la atenuación de la luz. Pero ese es un tipo de basura de ondas de mano que inventé y pertenece a la construcción de mundos más que a la física actual.

Incluso si el universo fuera finito, tenga en cuenta que el brillo de todas las estrellas en nuestra propia galaxia eclipsaría con creces al sol. Así que realmente no podemos tener un universo casi idéntico al nuestro a menos que la luz tenga una velocidad finita.

Además, tenga en cuenta que no sé nada sobre mecánica cuántica o cómo la velocidad de la luz afecta esa rama de la ciencia. Es posible que la mecánica cuántica actual no pueda existir con una velocidad de la luz infinita, lo que podría declarar significa que cualquier universo con una velocidad de la luz infinita es totalmente diferente al nuestro. 1 Sin embargo, considero que "similar a nuestro universo" significa cualquier universo con física de partículas que permita la formación de planetas, estrellas, vías neuronales, etc., a un nivel macro que un humano típico reconocería como similar. No me importa si el "oro" tiene más protones que el "hidrógeno", etc., y mucho menos efectos cuánticos.

Echa un vistazo a la física de la serie ortogonal de Greg Egan . Sus argumentos suponen un tiempo global fijo como Galileo, que no es la única solución. Egan tiene 4 dimensiones con el tiempo siendo una dirección relativa: velocidades infinitas (para el marco de referencia de los observadores ortogonales) y sin la catástrofe de Olber.
@JDługosz no es la paradoja de Olbers, ¡pero además hay algunas cosas bastante raras! Es un gran texto. Normalmente no soy fanático de la ciencia ficción, pero Orthogonal lo hizo por mí.
Como procrastinador repetitivo y fanático de toda la vida de la paradoja de Olbers, ¿cómo podría dejar pasar esta respuesta? (Además, proporciona previsibilidad). ¡+1!

Puedo sentir por su pregunta que está buscando una explicación simple y básica sin jergas. Le daré una oportunidad honesta y lo mantendré realmente simple y clásico. Soy un pensador clásico, así que ni siquiera tengo una explicación más compleja. Espero que los usuarios más calificados y acreditados no frunzan el ceño ante la respuesta.

Permítanme dividir la pregunta en dos partes:

(1) Por qué hay una cierta velocidad de las ondas electromagnéticas (que resulta ser c)

La velocidad de cualquier onda es propiedad del medio por el que viaja. Entonces, es propiedad del espacio vacío que las ondas electromagnéticas viajen a cierta velocidad (ni más, ni menos). Es una propiedad, no un límite. Si fuera un límite, entonces la luz (o EM) podría viajar a < c a través del espacio vacío. Pero viaja exactamente en c, en el espacio vacío. Entonces, es una propiedad. Si tiene problemas con esta explicación, entonces también debería tener problemas con el sonido que tiene una cierta velocidad a través del aire y necesita ir a un nivel más básico. Si acepta una cierta velocidad del sonido, espero que acepte esta propiedad también en el caso de las ondas EM. El valor de esta propiedad pasa a ser c. James Large también lo indicó en su respuesta a su pregunta del 21 de enero.

(2) Por qué ningún cuerpo material puede moverse más rápido que c

Esta es una consecuencia directa de (1) y, por lo tanto, resulta ser un límite. (Es límite porque los cuerpos pueden moverse a cualquier velocidad siempre que no exceda c)

Consideremos cómo aumentamos la velocidad de una masa: le aplicamos una fuerza. Por ejemplo, podemos hacer que un automóvil parado se mueva empujándolo con nuestras manos. Los electrones en nuestras manos y los que están en el carro (donde lo tocamos) se repelen entre sí y esa fuerza repulsiva provoca un aumento en la velocidad del carro. Suponga que está corriendo a su velocidad máxima y un automóvil pasa a su lado a 300 millas por hora. ¿Puedes aumentar la velocidad del coche empujándolo con las manos cuando pasa a tu lado? La respuesta es no (un ser humano normal no puede mover las manos a más de 300 millas/hora). Para aumentar la velocidad de un cuerpo en movimiento, la fuerza tiene que actuar sobre él más rápido que la velocidad del cuerpo en movimiento.

Cualquier fuerza que aplicamos para acelerar un cuerpo, la fuerza se ejerce finalmente sobre el cuerpo como una de las fuerzas fundamentales. Todas las fuerzas fundamentales viajan en c. Solo como ejemplo, la fuerza electromagnética se propaga a la misma velocidad que las ondas EM iec Para simplificar, aceptemos que todas las fuerzas fundamentales se propagan en c como una propiedad según (1) Por lo tanto, no pueden aumentar la velocidad de ningún cuerpo material que ya se esté moviendo a una velocidad de c a > c.

Tenga en cuenta que las fuerzas tienen que viajar más rápido que c a través del espacio para causar una velocidad mayor que c. Pero sabemos que se mueven en c. Entonces, incluso las fuerzas fundamentales se vuelven ineficaces para un cuerpo que ya se está moviendo en c.

Las fuerzas se vuelven ineficaces en la dirección del movimiento del cuerpo en c. Todavía son efectivos en otras direcciones y, por lo que el cuerpo puede ralentizarse, etc.

Por tanto, la velocidad de propagación de las fuerzas fundamentales es una propiedad (no un límite). Esta propiedad tiene el valor más alto en el espacio vacío que resulta ser c. Y nada puede propagarse más rápido que las propias fuerzas. ¿Qué hará que algo lo haga? La velocidad de las fuerzas (una propiedad como la velocidad del sonido) resulta ser un límite de velocidad para los cuerpos materiales. Lo cual no es un misterio en absoluto.

También puedo comentar que en los aceleradores de partículas, utilizan los campos eléctricos/magnéticos para acelerar las partículas y, obviamente, no pueden acelerarlas más rápido que c.

Por favor, LMK si esta explicación funciona para usted.

¿Entonces la respuesta es que la "velocidad de las fuerzas" no es mayor que c? Entonces, ¿por qué hay un aumento en la masa de las partículas cuando hay un aumento en la velocidad?
¿Cómo explicaría la fuerza de la gravedad sobre un fotón que cae directamente en un agujero negro? Si el fotón no va más rápido que c, ¿se debe a que la gravedad no le aplica ninguna fuerza (por ejemplo, no gana masa, energía, no se desplaza hacia el azul)?

Esta pregunta tiene una respuesta muy corta, pero utiliza la suposición que utiliza toda la relatividad. es decir, la velocidad de la luz es constante para todos los observadores.

Con base en esta suposición, es trivial demostrar que se observa un evento horrison a una velocidad de c cuando se trata de acelerar infinitamente.

Para responder por qué esta suposición es válida, debe observar la derivación de las ecuaciones de Maxwell que muestran que la velocidad de propagación de las ondas electromagnéticas es independiente del marco de referencia. La derivación de estas ecuaciones se basa en conceptos como "Conservación de la carga" y la ley de Faraday. No creo que la pregunta pueda responderse más profundamente que eso.

tldr: a través de la conservación de la energía => carga y observaciones empíricas de las interacciones electromagnéticas, se puede demostrar que la velocidad de la luz es independiente del marco de inercia. Esto se puede usar para probar que existe un límite de velocidad universal de c.

Como ha leído en otras respuestas, no es algo fácil de explicar. Parece tan contrario a la intuición. "Si quiero ir más rápido, ¿por qué no acelerar más?" o "Si mi velocidad es muy cercana a la velocidad de la luz, entonces disparo una bala, ¿no irá más rápido que la luz? ¿Por qué no?"

Relatividad

Comencemos por refinar lo que entendemos por "límite de velocidad". Digamos que estás en una nave espacial con una fuente de combustible infinita y un potencial de aceleración arbitrario. Estás en reposo (acoplado a una estación espacial) y quieres viajar a un sistema estelar a 10 años luz de distancia. ¿Cuanto tiempo te va a tomar? En esta hipotética nave espacial, podrías llegar allí en 10 MINUTOS, no en años (suponiendo que la aceleración no altere tu delicado cuerpo humano).

¿Pero no es esto una violación del límite de velocidad cósmico? ¡No! Técnicamente, no estás viajando más rápido que la luz. Desde su punto de vista, parece como si el espacio se aplanara y su destino se acercara a usted. Si alguien en la estación espacial estuviera mirándote embarcar, desde su punto de vista estarías viajando muy cerca de la velocidad de la luz, pero solo te verían llegar a tu destino dentro de 10 años en el futuro.

Ahora, demos un paso más e imaginemos que somos un rayo de luz que viaja por el espacio. Desde tu punto de vista, ¿cuánto tiempo pasa antes de que interactúes con algo? No hay tiempo en absoluto. Un rayo de luz se teletransportará instantáneamente desde la fuente a un destino sin el paso del tiempo. Pero, por supuesto, los mismos principios de relatividad se aplican aquí: un observador externo no experimentará esta teletransportación instantánea.

no relatividad

Ahora imagina un universo alternativo donde no hay límite de velocidad cósmica. Primero, la luz viajaría instantáneamente. Entonces, cuando miramos hacia el cielo, veríamos otras estrellas y planetas exactamente como son ahora. Podríamos viajar hacia y desde cualquier lugar del universo en una cantidad de tiempo arbitrariamente pequeña. ¿Parece realista verdad?

El problema es lo que sucede a menor escala. Imagine un proceso atómico, como el de nuestro Sol, en este universo hipotético. El núcleo del sol está a unos 15 millones de grados Celsius (recuerde: la temperatura está relacionada con la energía cinética). El sol tiene unos 4,6 segundos luz de diámetro.

Una velocidad relativista de la luz actúa como un acelerador, evitando que estas reacciones atómicas en cadena sucedan demasiado rápido. Ayuda a limitar qué tan caliente puede estar algo (al aumentar la masa de partículas que se mueven muy rápido, para evitar que se muevan demasiado rápido) y qué tan rápido pueden ocurrir las reacciones (fracciones de segundo para que la energía viaje versus instantánea, que es una GRAN diferencia). Esto podría significar que las estrellas explotan demasiado rápido para siquiera formarse. También podría significar que la energía que alimenta las reacciones escapa del sol demasiado rápido y no le da tiempo a reaccionar. No estoy seguro de qué sucedería, pero de cualquier manera los resultados son catastróficos.

Conclusión

El "límite de velocidad cósmica" es un aspecto importante de nuestro universo. Si bien es concebible diseñar un universo sin límite de velocidad relativa, los resultados no serían muy interesantes. Nuestra intuición nos dice que esto debería ser posible, pero a veces nuestra intuición sobre cosas que no entendemos completamente no es muy buena.

La pregunta de "¿por qué hay un límite de velocidad cósmico?" es tan fundamental como "¿por qué el universo contiene más materia que antimateria?" o "¿por qué existe el magnetismo?". Cabe replantear la pregunta, "¿Por qué vivimos en un universo con estas características?" O "¿Podríamos vivir en un universo con características diferentes?" Porque es posible que existan otros universos con características diferentes, y solo una pequeña porción de ellos puede sustentar vida. Si la vida humana existe, naturalmente lo hará en un universo que pueda sustentarla.

Esta publicación examina algunas de las consecuencias de tener un límite de velocidad, pero realmente no toca el por qué, excepto por el "clásico" tiene que ser así, de lo contrario no existiríamos. En cierto sentido, esa es una respuesta tan buena como cualquier otra, supongo.
Es como preguntar "¿Por qué la Tierra está a 93 millones de millas del Sol?" La respuesta es que nadie sabe exactamente. Simplemente sucedió de esa manera. La pregunta en sí es defectuosa, es completamente irrelevante. La verdadera pregunta es: "¿Podríamos siquiera existir para hacer la pregunta si la Tierra estuviera a otra distancia?"
Dijiste "Si alguien en la estación espacial te estuviera viendo embarcar, desde su punto de vista estarías viajando muy cerca de la velocidad de la luz" y no "verían" tu embarque hasta 10 minutos antes de que llegaras, ¿verdad? De lo contrario es absurdo. No veo luz "embarcar" hasta el momento en que llega.
@nocomprende No estoy seguro de lo que quieres decir. No estoy hablando del embarque ligero, estoy hablando del barco. Cualquier observador presenciaría que el barco tarda unos 10 años en llegar a su destino, pero la persona en el barco solo presenciaría el paso de 10 minutos.
"Veríamos otras estrellas y planetas exactamente como son ahora. Podríamos viajar hacia y desde cualquier lugar del universo sin ningún paso del tiempo". - ¿qué diablos? ¿Cómo implica lo primero lo segundo?
@immibis: Técnicamente, podría viajar a una velocidad infinita si se convirtiera en datos y los enviara a la velocidad de la luz. Pero necesitarías energía proporcional a la distancia al cuadrado para hacerlo, de modo que el receptor pueda entender la señal y reconstruirte, para que no sean distancias infinitas.
@MIchaelS Si eso cuenta como "viaje", lo cual es controvertido. Sin embargo, definitivamente no se está "moviendo".
@immibis, estás tomando ese párrafo demasiado literalmente. Estoy tratando de transmitir una explicación intuitiva de cómo sería un universo sin límite de velocidad. Si no hay un límite de velocidad, podría viajar arbitrariamente rápido, lo que significa que podría viajar cualquier distancia en una cantidad de tiempo arbitrariamente pequeña.
Un protón a 15 millones de kelvin no es relativista. No parece haber motivos para que el sol explote si nos deshacemos de la idea de la relatividad especial y permitimos la acción a distancia.
@LLlAMnYP Los cálculos de la parte posterior del sobre sugieren que los átomos se mueven a aproximadamente el 0,2% de la velocidad de la luz en el sol, con una dilatación del tiempo de aproximadamente el 0,0002%. Y, por supuesto, la radiación electromagnética viajaría instantáneamente. Las cosas ciertamente serían diferentes, pero estoy de acuerdo en que el resultado puede no ser una explosión.
La velocidad de la luz y la incapacidad de superar la velocidad de la luz no son necesariamente lo mismo. ¿Por qué la velocidad de la luz no puede ser la que es, pero uno puede viajar más rápido en una nave espacial arbitrariamente? Tomemos como ejemplo la velocidad del sonido y nuestra capacidad para superarla.
@Neil Son lo mismo. Ni siquiera deberíamos llamarlo "la velocidad de la luz", en realidad deberíamos llamarlo el "límite de velocidad relativo", y una propiedad de la luz es que viaja a esta velocidad. Y nuevamente, puede viajar más rápido que este límite, pero solo desde cierto punto de vista.
@JamesWatkins Ese es un argumento antrópico. Sería como cuando te preguntan por qué el cielo es azul y respondes: "Porque es la única forma en que puede ser". Esa puede ser incluso la respuesta correcta en este caso, pero te estás perdiendo el punto. Quizás la mejor pregunta aquí es: "¿ Por qué no pueden ser conceptos separados?"
@Neil Podrían ser. No hay una regla que diga "no se puede tener un universo sin límite de velocidad". Entonces, la pregunta general es "¿Por qué nuestro universo tiene uno?" Y eso es tan fundamental como preguntar "¿Por qué nuestro universo tiene magnetismo?" Fuera de las matemáticas pesadas, no hay una explicación satisfactoria.

En contraste con las otras respuestas, intentaré dar una respuesta simple.

Primero, tenga en cuenta que "Por qué" es una pregunta pobre para la ciencia moderna, ya que la ciencia moderna prefiere predecir "qué" sucederá con la mayor precisión posible utilizando "modelos" de lo que suponen que está haciendo la realidad.

La velocidad y el tiempo están fuertemente interrelacionados y efectivamente están bajo el mismo "límite de velocidad". Cuando vas lo más lento posible a través del espacio, entonces vas lo más rápido posible a través del tiempo y viceversa.

Nuestro universo observable parece tener un límite universal de "espacio-tiempo". Este límite es parte de las " constantes fundamentales " interconectadas de nuestro universo.

Lo interesante es que si alguno de ellos fuera alterado de manera significativa, nuestros modelos actuales predicen versiones muy diferentes de la realidad en las que es muy poco probable que la vida consciente llegara a existir (disculpe la especulación salvaje aquí) para poder preguntar esto. pregunta.

Esto se llama el principio antrópico.

Entonces la respuesta a tu pregunta

¿Sabemos POR QUÉ hay un límite de velocidad en nuestro universo?

es "sí, porque tenemos la suerte de vivir en un universo cuyo límite de velocidad aparentemente aleatorio permitió que apareciera vida consciente"

Puede haber muchos de esos universos, posiblemente con diferentes constantes fundamentales a las nuestras, posiblemente con formas radicalmente diferentes de vida sensible.

Las verdaderas preguntas de FÍSICA detrás de toda esta filosofía incluyen

  • "¿Hay otros universos?"
  • "¿Qué podemos saber acerca de ellos?"
  • "¿Cuáles son las distribuciones de probabilidad de las constantes fundamentales en esos universos?"
  • etc etc etc etc

Lamentablemente, no creo que se hayan propuesto experimentos científicos prácticos para probar estos modelos todavía.

Por lo tanto, este tema es más filosofía que física , por lo que su pregunta probablemente debería cerrarse como fuera de tema.

Redacción actualizada en respuesta al comentario.

FYI el principio antrópico es la especulación. Se utiliza como factor adicional para reducir el zoológico en el que se ha encontrado la teoría de cuerdas, sin una buena solución. Si el universo fuera diferente entonces la vida sería diferente, así de simple, no hay motivo para sentirse más afortunado de lo necesario. Tenga en cuenta la diferencia entre diferentes formas de vida vs no vida
Algunos dicen que el principio antrópico se expresa simplemente, lo siguiente: "El universo es tal y tal para que los cosmólogos puedan observarlo". No es bueno para mis estándares.
El principio antrópico no dice "este es el único universo posible en el que podría evolucionar la vida". Simplemente dice "si este universo no pudiera desarrollar vida, nadie estaría aquí para preguntar por qué no". Es un argumento en contra de que "el universo fue diseñado especialmente para nosotros", no en contra de la existencia de universos alternativos.
"Cuando vas lo más lento posible a través del espacio, entonces vas lo más rápido posible a través del tiempo y viceversa". ¿Existe alguna ecuación conocida que modele esta relación?
@JamesWatkins Sí, la ecuación de Lorentz describe la dilatación del tiempo en el contexto de la relatividad especial. ver simple.wikipedia.org/wiki/Time_dilation

Bien, es posible probar (teóricamente, y te aconsejo por Feigenbaum, 2008 ) que la homogeneidad e isotropía del espacio y la homogeneidad del tiempo conducen necesariamente a la existencia de un límite de velocidad.

Hagamos eso: imagina tomar el Universo y borrar (eliminar) todo tipo de objeto. Permaneces sólo con el espacio-tiempo mismo. En este espacio-tiempo no hay nada, no importa la energía.

Ahora bien: ¿es razonable pensar que, en un espacio-tiempo vacío, hay un solo punto privilegiado sobre el otro? No, entonces el espacio-tiempo vacío es homogéneo.

¿Es razonable pensar que, en un espacio-tiempo vacío, se privilegia una sola dirección sobre las demás? No, entonces el espacio-tiempo también es isótropo.

A partir de esos dos supuestos, Feigenbaum muestra la existencia de una velocidad límite. Por otro lado, también es interesante notar que en la teoría de Einstein, la existencia de una velocidad límite es un axioma. Sin embargo, esto es, en cierto sentido, innecesario. De hecho, asumiendo menos cosas (como solo homogeneidad e isotropía), se puede demostrar que tiene que existir una velocidad límite.

Ahora bien, el hecho de que ESA velocidad límite sea la de la luz es una cuestión que la teoría de Feigenbaum no muestra ni prueba. ¡Este hecho tiene que ser arreglado por un experimento!

Última advertencia

¿Qué es lo que realmente sigue siendo cierto en un espacio-tiempo lleno de materia? Todo lo que escribí anteriormente aún se ejecuta y es válido, pero solo localmente , es decir, en áreas muy pequeñas del espacio-tiempo y área por área.

Globalmente no tiene sentido afirmar/decir que existe una velocidad límite (porque el propio concepto de velocidad global está mal definido) y puede ocurrir que a veces se mueva con una velocidad superlumínica. Por ejemplo: las galaxias lejanas se alejan de nosotros con velocidades mucho mayores que la velocidad de la luz.

Gracias por tu respuesta, pero ¿cómo pasaste de un universo isotrópico y homogéneo a un límite de velocidad? (No seguí completamente tu argumento)
@LandosAdam esto se sigue de la relatividad galileana. Una vez que exiges que la física sea la misma en todos los marcos de referencia, rápidamente llegas a la exigencia de que, digamos, la velocidad de la luz sea la misma en todos los marcos de referencia, y eso conduce a la invariancia de Lorenz y la SR. Pero, ¿y si efectivamente hubiera un marco de referencia privilegiado? No muchas cosas se mueven a velocidades relativistas en relación con nosotros, así que no veo por qué un marco de referencia privilegiado debería romper muchas cosas.
sería bueno proporcionar la referencia real al artículo de Feigenbaum. Tenga en cuenta que la homogeneidad y la isotropía pueden usarse para derivar un límite de velocidad, pero no son básicas en el sentido de que el espacio-tiempo sin materia ni energía puede ser isótropo y homogéneo, pero no significa mucho. Por otro lado, CON materia y energía, la isotropía y la homogeneidad no necesariamente se cumplen. También se puede derivar el límite de velocidad del principio de la relatividad (nota, no las teorías de la relatividad ), que a su vez también se puede derivar de la causalidad.
He agregado el enlace al artículo de Feigenbaum sobre arxiv. Sin embargo, la respuesta también debe contener la parte crucial de la derivación matemática del documento; de lo contrario, es una respuesta de solo enlace.
Es posible que desee echar un vistazo a arxiv.org/abs/1209.0563 y a los documentos digitales de Einstein aquí y aquí . El espacio no es homogéneo y la velocidad de la luz varía.

Se puede decir que esto es sólo un resultado experimental. Que la luz (y otras señales/interacciones) no viajen con velocidad/velocidad infinita.

Uno puede dejarlo ahí y decir que es así.

También se puede decir, mire, si toma esa variable y hace esa transformación (por ejemplo, rapidez ), se puede agregar ad-infinitum, por lo que la pregunta es sobre la variable correcta a usar. Aunque esto simplemente pasa por alto la pregunta real en lugar de abordarla.

Adoptaré otro enfoque y dirigiré la pregunta directamente al corazón.

La velocidad finita de transmisión de señales (o interacción) es un requisito básico para que se mantenga la causalidad .

De lo contrario, si la transmisión de señales puede ser infinita, un efecto puede superar a su propia causa y los bucles causales resultantes hacen que la causalidad deje de ser causalidad. Algo que también es un hecho experimental pero de nivel aún más básico. En este sentido, esta es la respuesta a la pregunta.

Uno puede llevar esto aún más lejos y derivar una transmisión de velocidad finita (límite superior) directamente de consideraciones termodinámicas, algo que está fuera del alcance de esta pregunta, pero menciónelo para un estudio adicional.

Sin embargo, la velocidad finita de transmisión de la señal no es necesariamente equivalente al postulado de la Relatividad Especial de que la velocidad de la luz es esta velocidad máxima (y constante) disponible.

Uno puede tener muchas velocidades de transmisión finitas diferentes menores o incluso mayores que la velocidad de la luz dependiendo del proceso en estudio.

De hecho, hay algunas investigaciones sobre la transmisión de señales más rápidas que la luz a través del entrelazamiento cuántico . Pero dejaré esto en este punto.

Pero la causalidad podría ser el efecto de tener un límite superior en la velocidad con la que la información puede viajar como dijiste. Entonces, no puede usarlo como un argumento para explicar por qué existe el límite porque está hablando en retrospectiva.
@LandosAdam, hmm, uno debería tomar como básico lo que parece más básico. La causalidad es un concepto más profundo y extenso que excede el dominio de la mecánica y las velocidades. En este sentido se toma como más básico. Pero puedo desviarme si lo contrario se da con la misma generalidad :)
Pero estoy hablando solo de causalidad para las cosas que suceden en nuestro universo DEBIDO a la velocidad límite, no de causalidad en su sentido general. O dicho de otro modo, en un universo alternativo donde no habría límite superior, no existiría la causalidad para cosas como la que mencionas. (La causalidad aunque en su forma general existiría)
@LandosAdam, hmm, ¿puedes aclarar esto? creo que no serás capaz de hacerlo. En cualquier caso, si se demuestra que los dos son equivalentes (o conceptos isomorfos, digamos), entonces no hay problema en cuál usar. Si no, entonces uno será más general que el otro. Hasta ahora la causalidad parece ser
En ese universo alternativo, la causalidad con señales (de su propio ejemplo) ni siquiera tendría un significado, una existencia. Simplemente no les importaría que el efecto supere su propia causa porque su universo y la forma en que funcionaría serían fundamentalmente diferentes. Creo..! ¡No puedo aclararlo más ya que ni siquiera estoy seguro de lo que estoy diciendo y podría ser anticientífico en este momento (con todas las suposiciones)! De todos modos, entiendo tu punto, pero tengo la sensación de que la causalidad de la que hablas es producto del límite de velocidad y ciertamente no al revés.
Sí, eso es exactamente lo que yo también entiendo. La mayoría de las estructuras e interacciones que nos rodean están formadas por EM. Si EM se vuelve infinitamente rápido, todo esto se vuelve infinitamente rápido. Y nose si el resto de fuerzas aguantarán el asunto. ¿Es el tiempo... electromagnético?
@LandosAdam, ya veo, ¿qué contaría como una respuesta a por qué el límite de velocidad es finito? ¿Tienes una idea o propuesta? Sería bueno escuchar. Tenga en cuenta que en la respuesta se menciona que la termodinámica puede derivar la transmisión de señales de velocidad finita sin otras suposiciones (¿tal vez esto es mejor? desde esa perspectiva).
No, no tengo una idea, solo estoy tratando de descifrar las ideas de los expertos (porque aún no soy un experto), pero estoy tratando de hacerlo sin engañarme a mí mismo al "creer" cada argumento que uno podría presentar. . Simplemente me parece que muchas de las cosas en las que piensa la mayoría de la gente para responder a mi pregunta se basan en la existencia del límite en lugar de explicar por qué existe (si hay un por qué). Lo siento si llego a ser crítico, pero solo quiero argumentar de una manera constructiva.
@noncom, no estoy seguro de si el tiempo (¿o es la duración ?;)) es electromagnético, pero la mayoría de las interacciones (excluyendo la gravedad por el momento) son de hecho generalizaciones del electromagnetismo (también conocida como teoría de Yang-Mills)
@LandosAdam, no hay ningún problema. Después de todo, esta es tu pregunta. Vea una pregunta mía similar aquí (en la misma línea, digamos). Yo mismo tengo una respuesta, pero quería escuchar lo que otros podrían decir.
Supongamos que el universo siguiera la relatividad galileana. Entonces, ¿de qué manera se violaría la causalidad? Ya sigue la relatividad galileana aproximadamente a bajas velocidades, pero no violamos la causalidad a bajas velocidades.
Si la información pudiera transmitirse instantáneamente entre objetos a una distancia arbitraria, ¿cómo aboliría eso la causalidad? Si no pudiéramos determinar el orden de causa y efecto debido a que no tenemos una buena brecha de duración entre los dos, ¿detendría eso la causalidad o solo interferiría con nuestra percepción de la causalidad?
@Corey: No se viola la causalidad. La gente está demasiado obsesionada con las ecuaciones SR que predicen tales violaciones en función de lo que sabemos actualmente sobre nuestro universo. La información aún no sería instantánea. Las estructuras físicas estarían hechas de partículas masivas que tendrían velocidades finitas, por lo que siempre habría algún retraso entre la transmisión, la recepción, la comprensión, la respuesta, etc. Además, el efecto nunca podría llegar a la causa en ningún momento antes de que ocurra la causa. por lo tanto, la causalidad aún no se viola.
@MichaelS Simplemente me confunde cuando las personas afirman que la causalidad se desmorona en un conjunto de condiciones que en realidad no parecen conducir a ningún conflicto con la causalidad. ¿Tal vez es cómo piensan sobre el tiempo?
@immibis, el argumento es discutible ya que la relatividad galileana o no no excluye la transmisión de señales finitas. Confundes la transmisión de señales finitas con la velocidad máxima y constante de la relatividad especial. En el sentido de esta publicación, la relatividad especial es solo eso, un caso especial. Ni mas ni menos. De hecho, a través de la termodinámica, se pueden derivar velocidades de señal finitas (o velocidades características sin asumir la relatividad especial).
@Corey, mira mi comentario anterior. La distinción entre causalidad y percepción de causalidad no está bien expresada simplemente porque si algo existe, hay alguna manera en que uno puede experimentarlo (no necesariamente de una manera determinada). Si no hay absolutamente ninguna forma de que uno pueda experimentar la causalidad (o cualquier otra cosa), entonces si existe o no es lo mismo, nada y sin sentido. Punto claro, espero que esto no te confunda.
@MichaelS, vea mis comentarios anteriores, espero que respondan a cualquier objeción que pueda tener
@NikosM. Es una analogía. Si se mantuviera la relatividad galileana, una velocidad finita de la luz no violaría la causalidad, como tampoco la viola la velocidad finita de unas pelotas de goma. Simplemente significaría que la luz no era lo más rápido posible.
Todos aquí siguen aludiendo a la RS cuando se les hace la pregunta "¿y si se mantiene la relatividad galileana?" Pero, ¿y qué, si tenemos que tener un marco de referencia absoluto y la velocidad de la luz no es la misma para todos los observadores? ¿Entonces algo se rompería fundamentalmente en el universo? ¿No se formarían las estrellas y la vida? Hay muchos procesos que no requieren SR o GR para describirlos.
@LLlAMnYP sí, todo se rompería. El electromagnetismo y la luz necesitarían equivalentes provistos de una manera completamente diferente. Tener fenómenos de nivel macro similares requeriría una implementación de bajo nivel totalmente diferente. Los detalles de la fuerza débil , que hace brillar a las estrellas, también emergen de las simetrías involucradas en el nivel más bajo. Cambiar el grupo de simetría del espacio-tiempo saca la alfombra debajo de él. ¡Piense en intentar ejecutar el código de máquina x86 en un procesador ARM!

No es particularmente inusual que los sistemas físicos tengan límites de velocidad.

Considere la clásica cuerda elástica vibrante, definida por la ecuación

2 t 2 y ( X , t ) = a 2 X 2 y ( X , t )

Usando esa ecuación, puedes ver que una pequeña perturbación en una parte de la cuerda se propagará hacia afuera a una velocidad particular. De hecho, verá que la velocidad a la que cualquier perturbación puede viajar a lo largo de la cuerda está limitada por esa velocidad.

Puedes dar sentido intuitivo a esta velocidad imaginando que la cuerda está hecha de pequeñas cuentas unidas por hilos elásticos, y la señal tiene que propagarse yendo de cuenta en cuenta, lo que limita su velocidad.

Verá un fenómeno similar en las ecuaciones diferenciales para un sólido elástico 3D (como un cubo de gelatina). También las ecuaciones diferenciales para una señal eléctrica en un cable, o una onda electromagnética en el espacio, o una onda sonora en el aire.

Casi cualquier sistema que pueda describirse mediante una ecuación diferencial que relacione la tasa de cambio en el tiempo con una propiedad local como la derivada o la densidad terminará teniendo un límite de velocidad. Y es bastante común que los sistemas físicos se comporten de esa manera, porque la mayoría de las cosas en el mundo están formadas por partes más pequeñas, y el comportamiento macroscópico del sistema se puede analizar en términos del comportamiento de las partes más pequeñas.

Me doy cuenta de que esto no responde en absoluto por qué . Solo quiero señalar que tener un límite de velocidad no es algo tan inusual o sorprendente.

En pocas palabras, es la forma en que la naturaleza preserva la causalidad . De Wikipedia :

"Por otro lado, si las señales pudieran moverse más rápido que la velocidad de la luz, esto violaría la causalidad porque permitiría enviar una señal a través de intervalos similares al espacio, lo que significa que, al menos para algunos observadores inerciales, la señal viajaría hacia atrás en el tiempo. Por esta razón, la relatividad especial no permite una comunicación más rápida que la velocidad de la luz".

Si no hubiera un límite de velocidad, sería posible todo tipo de paradoja que implicara la violación de la causalidad (¿quizás has oído hablar de la paradoja del abuelo ?).

Pero, esta respuesta, como la mayoría de las otras, tiene que ver con el efecto de la limitación y no con la razón detrás de ella.
Entonces no creo que sea posible una respuesta. Esto es como preguntar "¿por qué la fuerza es igual a la masa por la aceleración?", o "¿por qué la mecánica cuántica se rige por la ecuación de Schroedinger?"...
Bueno, realmente no pensé que habría una respuesta. Solo pregunté en caso de que haya una respuesta porque estoy en mis primeros pasos como físico, ¡así que hay muchas cosas que no sé y no puedo derivar (si son derivables)! :)
¿Por qué tu respuesta es tan baja? ¡Tienes 100% de razón! Necesitamos un límite de velocidad (y no solo un límite sino un límite invariante) para que la causalidad pueda ser preservada en nuestras teorías de la relatividad... Para Galileo era nuestra velocidad a través del tiempo, era constante, invariante e igual a 1s/s. Para Einstein es nuestra velocidad a través del espacio-tiempo, su constante, invariante e igual a 1ls/s.. Solo necesitamos un límite de velocidad para preservar la causalidad.. ¿Por qué necesitamos hacer eso? Porque ASUMIMOS que el mundo es determinista (al menos a gran escala), por lo que construimos una teoría determinista para explicar el mundo tal como lo entendemos.
"¿Es nuestro mundo determinista?" es la pregunta de seguimiento, y hasta ahora, es filosófica, por lo que no se puede responder aquí. La "razón" detrás de la limitación es preservar la causalidad... Nadie en la época de Galileo preguntó nunca "¿por qué tengo que envejecer 1 segundo cada segundo? ¿No puedo envejecer más rápido o más lento?"... No, porque eso era No es poco intuitivo, así que nadie cuestionó eso... Nadie hasta Einstein, quien entendió que ¡SÍ! De hecho, ¡puedes envejecer más rápido o más lento en el tiempo! ¡Pero en el espacio-tiempo TIENES que ir siempre a 1ls/s, de lo contrario romperás la causalidad! ¡O al menos, nuestras teorías no podrán rastrear!

La pregunta en el encabezado era

¿Sabemos por qué hay un límite de velocidad en nuestro universo?

Luego hubo una amplificación.

Esta pregunta es sobre por qué tenemos un límite de velocidad universal (la velocidad de la luz en el vacío). ¿Hay una ley más fundamental que nos diga por qué es esto? No estoy preguntando por qué el límite de velocidad es igual a c y no otra cosa, sino por qué hay un límite.

Creo que, de todas las respuestas, @Anna_v ha estado más cerca de responder la pregunta.

Actualmente la respuesta a la pregunta es “No”.

La idea de que existe un límite de velocidad universal proviene de las observaciones del Universo.
Estas observaciones dan como resultado algunas teorías que pueden usarse para hacer predicciones sobre el Universo.
Varias de estas teorías que son buenas para hacer predicciones tienen la idea de que existe un límite de velocidad universal.
Dicho de otra manera, el límite de velocidad universal es un postulado útil porque hace que algunas de las teorías "funcionen".
Las teorías actuales no pueden predecir todo lo que ha sucedido y sucederá, por lo que los científicos buscan mejores teorías.

Se desconoce si existe una teoría que explique por qué existe un límite de velocidad universal.

En la actualidad tal teoría no existe.

Una respuesta física:

Cuando un cuerpo cargado eléctricamente se mueve en relación con un observador, el observador puede medir un campo magnético inducido. La energía almacenada en este campo magnético tiende al infinito mientras que la velocidad del cuerpo en movimiento se acerca a c.

Esto no es del todo intuitivo con la relatividad general... ¿También está diciendo que dos cuerpos cargados que se dirigen entre sí a .9c tendrán un campo magnético inducido más grande para un observador estacionario presente en el punto en que se encuentran que cada viajero observa? (¿Es solo 2x para el observador externo, o se vuelve loco porque la velocidad delta entre los dos objetos que se acercan es 1.8c?)
@BenPen, todo depende del movimiento del observador. Y el campo magnético es diferente en diferentes sistemas de inercia, aunque todos los sistemas están "alojados en el mismo universo".

He notado que cuanto más se acerca uno a las teorías físicas fundamentales, las que describen las interacciones más básicas en nuestro universo, más empiezan a parecerse todas las ecuaciones a transformaciones de coordenadas. A veces, estas coordenadas están en espacios abstractos, los grupos del modelo estándar de la física de partículas y los espacios de Hilbert de la mecánica cuántica, pero, en última instancia, la física es una descripción del movimiento de las cosas.

Para localizar algo en el universo, necesitas tanto la posición como el tiempo. Ahora, incluso si está sentado quieto mientras lee esto, se está moviendo a través del tiempo. La velocidad a la que te mueves en el tiempo es de un segundo por segundo según tu propio reloj, pero no todo el mundo estará de acuerdo con eso. Averigüemos qué tan rápido vas según cualquier observador.

Vamos a enviarte en un viaje al sistema estelar Alpha Centauri a una gran fracción de la velocidad de la luz. Un residente en su destino observa su viaje y ve que recorrió una distancia de d (alrededor de 4 años luz). Según el reloj que llevas puesto, has envejecido un tiempo t , que es menor que el tiempo que el Alpha Centaurian midió su viaje debido a la dilatación del tiempo. Para encontrar su viaje total a través del espacio-tiempo, podemos combinar su viaje en dos dimensiones con el teorema de Pitágoras:

X = d 2 + t 2 .
La distancia total que recorriste en el espacio y el tiempo es X ; la distancia que recorriste en el espacio es d ; y la distancia que recorriste en el tiempo es t (lo que equivale a decir cuánto envejeciste). El problema de esta ecuación es que d y t están en diferentes unidades: metros y segundos. Afortunadamente, la relatividad de Einstein proporciona un factor de conversión: la velocidad de la luz. Entonces, la ecuación debería decir:
X = d 2 + ( C t ) 2 .

Ahora, la distancia que viajó es igual a la velocidad de la nave espacial multiplicada por el tiempo del viaje medido por el Alpha Centurion (la distancia medida en el marco de reposo de dos puntos se denomina distancia adecuada. El tiempo adecuado se mide con un reloj en reposo con la entidad que está siendo cronometrada, es decir, su reloj). Llamemos al tiempo transcurrido en Alpha Centauri t α .

X = ( v t α ) 2 + ( C t ) 2 .
podemos relacionarnos t y t α con la ecuación de dilatación del tiempo:
t = t α γ = t α 1 ( v C ) 2
dónde γ es el factor relativista que aparece en casi todas las ecuaciones relativistas. Darse cuenta de t α es más pequeña que t para reflejar el envejecimiento más lento que experimentan los objetos que se mueven rápidamente (usted).

Entonces, ahora tenemos

X = ( v t α ) 2 + ( C t α 1 ( v C ) 2 ) 2 .
Simplificando:
X = v 2 t α 2 + C 2 t α 2 ( 1 ( v C ) 2 ) = t α v 2 + C 2 ( 1 ( v C ) 2 ) = t α v 2 + C 2 v 2 = t α C 2 = C t α .
La distancia total que recorriste a través del espacio y el tiempo es igual a la velocidad de la luz multiplicada por el tiempo de viaje. Esto es cierto sin importar cuál sea su velocidad. Por lo tanto, cuando tomas tu movimiento a través del espacio y el tiempo juntos, ¡siempre te estás moviendo a la velocidad de la luz! Pensar que diferentes objetos viajan a diferentes velocidades ignora su movimiento a través del tiempo. Entonces, la velocidad de la luz no es solo una velocidad máxima. También es una velocidad mínima. Se podría decir que es la única velocidad.

Una consecuencia de esto es que cuanto más rápido te mueves por el espacio, más lento te mueves por el tiempo y viceversa. Puede imaginar esta situación como si estuviera conduciendo un automóvil sin pedal de acelerador ni pedal de freno, solo un volante. Siempre viaja a la misma velocidad. Si desea conducir hacia el este, debe sacrificar algo de velocidad en dirección norte. De la misma manera, si quieres moverte por el espacio, debes sacrificar algo de velocidad a través del tiempo. De hecho, las matemáticas funcionan de la misma manera para la relatividad si imaginas un eje como espacio y el otro como tiempo, como lo hice en la derivación anterior.

Ahora bien, ¿existe una razón fundamental para responder por qué esto es así? Lo mejor que se me ocurre es observar que no tenemos control sobre la velocidad a la que envejecemos. Un año para ti es exactamente el mismo año para mí (a menos que pronto se inventen algunas naves espaciales avanzadas). Si el tiempo no es tan diferente del espacio, como parece ser en nuestro universo, entonces, al igual que el tiempo, viajar por el espacio también estaría limitado a cierta velocidad. El hecho básico sobre nuestro universo que establece el límite de la velocidad de la luz es la interdependencia del movimiento a través del espacio y el tiempo.

Eres demasiado bajo para una respuesta tan elegante :/

¿Por qué hay un límite de velocidad en nuestro universo? Esto podría tener algo que ver con el principio de localidad en la física. Tenga en cuenta que en el universo de Fredkin como un autómata celular, siempre hay un límite de velocidad, para cualquier patrón emergente (solo un ejemplo). Entonces, la existencia de un límite de velocidad en nuestro universo es un respaldo (consecuencia) del principio de localidad en la física. Como nota al margen, los argumentos cuánticos de no localidad (basados ​​en experimentos de entrelazamiento cuántico) deben explicarse en términos de sistemas caóticos sincronizados, pero sin rechazar el principio de localidad.

Se necesita alguna constante c con las dimensiones de una velocidad porque los impulsos no conmutan y, por lo tanto, los aumentos deben realizarse en radianes adimensionales (matemáticos). La constante c convierte las velocidades a radianes. La constante c no puede ser infinita porque eso haría que los impulsos se conmutaran.

Darle a un objeto una velocidad (impulso) en la dirección x no conmuta con el impulso en la dirección y. Impulsos y rotaciones obedecen empíricamente a la definición de grupo. La relatividad especial descubrió que los impulsos son miembros del grupo de Lorentz no abeliano. Para v C 1 es cierto que

Aumentar ( v X C ) Aumentar ( v y C ) Aumentar ( v y C ) Aumentar ( v X C ) = Rotación z ( v X C v y C ) .

debe haber una constante C con dimensiones de velocidad para hacer que los parámetros de impulso sean radianes adimensionales para que su producto (el ángulo de rotación sobre el z -eje) también puede estar en radianes adimensionales. esta bien eso r a d i a norte s 2 = r a d i a norte s como lo demuestran los términos en la expansión de la serie de potencias de pecado ( θ ) . Es una tontería hacer una rotación sobre el z -eje por ( metro s ) 2 .

Si C , entonces los impulsores viajarían y ya no serían parte del grupo Lorentz. La constante c es similar a la constante a = ( 180 π ) d mi gramo r mi mi s que se utiliza para convertir ángulos Θ de grados a radianes. El grupo de rotación (que es un subgrupo del grupo de Lorentz) no es abeliano

Rotación ( Θ X a ) Rotación ( Θ y a ) Rotación ( Θ y a ) Rotación ( Θ X a ) = Rotación z ( Θ X a Θ y a ) .

El " a " es necesario. Sería una tontería hacer una rotación sobre el z -eje por [ grados ] 2 . Si a , luego las rotaciones cambiarían y ya no serían parte del grupo de Lorentz. Si las rotaciones conmutaran, nuestro mundo sería muy diferente. No existiría tal cosa como rotar un objeto en un ángulo distinto de cero y hacer que vuelva a su orientación original. Además, el momento angular no se cuantificaría y las partículas no tendrían espín.

En resumen, c (y a) son necesarios y deben ser finitos porque los impulsos (y las rotaciones) son parte del grupo de Lorentz no abeliano. Este grupo es donde el parámetro de refuerzo [ rapidez ] = bronceado 1 ( v C ) proviene de la respuesta de Kostya.

Buen enfoque del tema, pero plantea la conclusión antes de llegar a ella, implícitamente. Suponiendo que las simetrías del espacio-tiempo para satisfacer el grupo de Lorentz equivalen a la constancia de la velocidad de la luz...
Estamos acostumbrados al camino histórico hacia la RS en el que primero se descubrió la invariancia de la simetría del espacio-tiempo de la métrica de Minkowski y se necesitaba ac para dar a ct la misma dimensión que x. Aprendimos a continuación que estas transformaciones de simetría forman el Grupo de Lorentz como derivadas de esa simetría.
... Sin embargo, supongamos que descubrimos empíricamente que el producto de los impulsos no solo agregaba velocidades, sino que causaba algo de rotación y que los impulsos no conmutan. Al ver cómo se comportan los productos, descubrimos que podemos identificar los elementos del grupo de O(3,1) con rotaciones y aumentos y lograr que nuestros símbolos en una hoja de papel copien las observaciones del mundo real. Solo entonces vemos el elemento de línea. d s 2 = d X 2 + d y 2 + d z 2 ( C d t ) 2 se deja invariable por el grupo. Ahora podríamos decir que el grupo es la razón fundamental por la que se necesita ac. La invariancia de simetría de la métrica es entonces el resultado secundario.
¿El grupo de qué? ... De transformaciones espaciales y/o temporales que mapea sistemas físicos a sistemas físicos isomórficamente (con respecto a la física). Esto es lo que quise decir con simetrías del espacio-tiempo. Ahora, mi punto inicial es que asumir (u observar) esta estructura implica directamente la constancia de la velocidad de la luz. La pregunta sobre el límite de velocidad ahora se convierte en por qué la realidad exhibe esta estructura de grupo particular.

Bien, estamos hablando de velocidades y del hecho de que hay un límite para la velocidad. Por lo tanto, estamos hablando de movimiento. El movimiento contiene dos variables. Uno es la velocidad, y el otro es la distancia. Las variables van de cero a infinito. Por lo tanto, para ver la imagen más grande posible con respecto al movimiento, uno empujaría naturalmente ambas variables al infinito.

Viajar a una "velocidad" infinita significa viajar a través de cualquier distancia, en tiempo cero. Es decir, uno podría viajar del punto A al punto B en, digamos, 1 minuto, pero eso también significa que podría viajar más rápido y completar el viaje en menos de, digamos, solo 1 segundo. Cuanto más rápido vaya, menos tiempo se requiere. Estas son velocidades finitas. Pero si viaja del punto A al punto B en muy poco tiempo, entonces no hay forma de que pueda superar esta velocidad en particular. Esta es la velocidad infinita.

Próximo. Viajar a través de una "distancia" infinita significa que seguirás para siempre, ya que una distancia infinita no tiene fin. Por lo tanto, si combina los dos y viaja a través de una distancia infinita a una velocidad infinita, esto significa que continuará para siempre, en muy poco tiempo . Holísticamente, esto simplemente no es posible. Sin embargo, relativistamente, es posible, y es posible dado que los dos extremos se separan.

En un posible extremo, puede moverse a través del espacio pero no se moverá a través del tiempo. En el otro extremo posible, te estarás moviendo a través del tiempo pero no del espacio. Para que esta transición sea posible, si ganas en uno, debes perder en el otro. Por lo tanto, no puede tener ambos extremos al mismo tiempo. Por lo tanto, a su vez deben estar englobados dentro de un finito para causar este fenómeno de ganancia y pérdida.

A su vez, usted puede estar moviéndose a través del espacio pero no a través del tiempo, mientras que para aquellos que lo observan, el tiempo sigue corriendo y, por lo tanto, aunque el tiempo esté detenido para usted, el tiempo puede continuar para siempre, en otro lugar. Así puedes continuar para siempre, en muy poco tiempo.

Por lo tanto, el movimiento finito a través del espacio-tiempo es un requisito para que el movimiento sea posible. Por lo tanto, existe un límite finito para la velocidad de movimiento en el espacio.

Si luego analiza el resultado de este fenómeno, descubre de forma independiente la Relatividad Especial y deriva de forma independiente todas sus ecuaciones matemáticas. Vea este video para verificar.

Por favor, si vota a favor, proporcione una explicación. Después de todo, mi pensamiento lógico relacionado con un simple análisis de "Movimiento" me había llevado a mi descubrimiento independiente de los fenómenos SR y a mi derivación independiente de las ecuaciones SR, y lo hice de una manera que nadie más había pensado. hasta el momento. Si mi lógica es incorrecta, entonces no hay límite de velocidad y las ecuaciones de SR también son incorrectas. De acuerdo, no he recibido educación física previa en absoluto. Pero he aprendido que si descubres la RS por tu cuenta, se te considera una persona atrasada. Esto puede estar relacionado con cualquier voto negativo que aparezca.
No soy el votante negativo, pero supongo que la idea general es que su explicación es confusa (no lo suficientemente rigurosa como para derivar un reclamo). He aquí por qué: decir que ''[Si] viajas a través de una distancia infinita a una velocidad infinita, esto significa que continuarás para siempre, en muy poco tiempo'' es falso. Está cancelando infinitos sin especificar el comportamiento asintótico de sus cantidades. equivale a decir 0 = , que no está definido. En segundo lugar, implícitamente asumes la conclusión cuando estableces tus dos extremos, al tomar tiempo para ser similar a una dimensión espacial.
G. Bergeron - Bueno, siempre puedes ver mis videos de YT y ver cuál es el resultado. Son accesibles a través de mi perfil de red.

El punto de partida del argumento de Einstein fue que las coordenadas del espacio-tiempo no son un adelanto físico, sino que se establecen a partir de los procedimientos de medición física. Si asumimos el principio general de la relatividad

  • Las leyes locales de la física son las mismas independientemente de la materia de referencia que utilice un observador en particular para cuantificarlas.

entonces todos los observadores establecerán coordenadas de la misma manera, lo que significará que una velocidad máxima (si la hay) será la misma para todos los observadores. O hay, o no hay, una velocidad máxima en la naturaleza. Descartando el argumento de que la ausencia de una velocidad máxima contradice la observación, podemos observar que todos los procesos físicos toman tiempo. La ausencia de una velocidad máxima implicaría la posibilidad de una acción instantánea a distancia (al menos en un límite), que Newton describió como

un Absurdo tan grande que creo que ningún Hombre que tenga en Asuntos filosóficos una Facultad competente de pensar puede jamás caer en él .

En GR, la velocidad de la luz no es constante, varía con la curvatura del espacio-tiempo. Entonces, la constancia de esta velocidad universal depende de que el espacio-tiempo tenga una curvatura constante. Lo cual no es así, pero esta es una aproximación localmente útil, y para abordar la intención del OP, de ahora en adelante supondremos que el Universo es un espacio de curvatura constante. En cualquier caso, sabemos que la velocidad de la luz es menor cuanto mayor es la curvatura, por lo que si buscamos el límite, tenemos que considerar el caso de curvatura cero constante, ya que en cualquier otro lugar será menor.

Ahora, por simplicidad, suponga que esta curvatura es cero.

Se observa experimentalmente que la masa es equivalente a la energía, por lo que tienen las mismas unidades. Pero la masa adicional producida por la energía cinética de una velocidad v es 1 2 metro v 2 entonces v debe ser adimensional. Por lo tanto, existe un sistema de coordenadas para el espacio-tiempo en el que las coordenadas x tienen las mismas unidades que las coordenadas t. Dado que la variedad es completamente plana, prácticamente euclidiana (excepto por el -1 en la firma de la métrica), podemos elegir un sistema de coordenadas que, hablando ingenuamente, sea el mismo en todas partes. Entonces, una dirección similar al espacio se puede rotar a una dirección similar al tiempo de la misma manera, uniformemente, en todas partes. (Esto puede sonar como SR, pero aún no es SR. Esto es simplemente un análisis dimensional más geometría simple más ese hecho experimental de la equivalencia de masa con energía). Pero luego tenemos una velocidad universal, esta misma conversión entre la coordenada x y la coordenada t.

Hasta ahora, esto no dice que la velocidad sea un límite de velocidad, ni que tenga que ver con la luz. Pero es canónico e intrínseco y "físico" ya que depende de la relación de conversión entre masa y energía.

El siguiente paso es deducir que se trata de un límite de velocidad universal. Eso se hace como de costumbre, ya que la aceleración aumenta la masa del objeto y, por lo tanto, se aplica un "rendimiento decreciente" exactamente cuantitativo.

Así que todo lo que queremos se deriva de la relación de Newton entre masa, energía cinética y velocidad, más el hecho experimental de la equivalencia masa-energía.

Nota: William Davidon publicó en alguna parte una nota que muestra cómo todo SR se deriva de la equivalencia masa-energía. No lo leí, pero solo escuchar el hecho de que lo hizo, obviamente, me dio una pista sobre esto. Así que hay que reconocer esa "prioridad".

No puede haber una razón filosófica muy básica por la que la masa tenga que ser equivalente a la energía, ya que la física teórica es posible de forma galileana, donde no es cierto. Por otro lado, filosóficamente, uno siempre podría considerar el caso de Galileo como incluido en este marco en el sentido de que es una constante universal y también un límite de velocidad universal, con los mismos derechos que 1. (Es cero como un límite de velocidad universal que nunca podría aceptarse en física... ni siquiera en teoría).

No publique la misma respuesta dos veces , pero vote para cerrar las preguntas duplicadas en lugar de copiar su respuesta.
Estimado José F. johnson. A menudo está mal visto publicar respuestas casi idénticas a publicaciones similares. En tales casos, a menudo es mejor simplemente marcar/comentar las preguntas duplicadas, para que puedan cerrarse.

La cantidad de masa que tiene un objeto = cuán resistente al cambio en el momento es ese objeto. Cuanta más masa, más energía se requiere para cambiar el impulso. Dado que los fotones no tienen masa, no tienen resistencia a los cambios de momento. Los fotones son esencialmente la velocidad a la que la energía se propaga a través del universo. En cuanto a por qué es c, así es como funciona el universo. No existe una ley que diga que algo no puede moverse más rápido, pero si consideramos que el mundo físico es la dualidad de energía/masa, entonces nada físico puede ir más rápido que la luz.

La razón por la que hay un límite de velocidad en nuestro Universo es que no tenemos nada (ninguna energía/fuerza) que pueda ayudarnos a mover algo más rápido que la velocidad más alta disponible para nosotros. ( C ) .

En otras palabras, si la velocidad de C eran 2 10 8 metro s (o 4 10 8 metro s ), entonces este sería el límite de velocidad del Universo. Lo mejor que podríamos hacer sería igualar la velocidad de C .

La luz viaja más rápido en el universo debido a su velocidad pero también a su trayectoria. Pierre de Fermat sostenía que "la luz viaja entre dos puntos por el camino del tiempo más corto". Esto puede llamarse el Principio del Tiempo Mínimo. Luego está el Principio de Acción Mínima. Citando a Pierre Louis Maupertuis "la naturaleza es ahorrativa en todas sus acciones". ¿Puedes pensar en un ejemplo en la naturaleza donde se desperdicien recursos/energía? Sugeriría con cautela que la naturaleza no necesita ningún movimiento más rápido que la luz y, por lo tanto, no lo ha proporcionado.

Un voto negativo significa que la respuesta es correcta (en este caso)

La velocidad de la luz se aplica a algo más que la luz, nada puede propagarse más rápido, incluidas las fuerzas que actúan sobre la materia. Sin embargo, hay una respuesta no tradicional de por qué. La velocidad de la luz es una velocidad máxima porque todos viajamos en una onda que transporta un medio, el espacio-tiempo. La luz, como onda en el medio, viaja a velocidad constante en él; y como todo lo demás en un viaje suave en un "fluido", nos movemos más lento que una onda que viaja en el fluido del espacio-tiempo, como se observa con objetos en agua o aire u ondas en sólidos. (Los sólidos son un caso especial, ya que solo las ondas viajan a través de ellos).

La analogía sugiere que puede que no haya un límite de velocidad absoluto , pero todo lo que hemos observado viaja a través del "fluido" del espacio-tiempo y no puede usarse para acelerar más allá de la velocidad de la luz, por lo que estamos atascados. Clásicamente hay dos casos de Aceleración.

1) Lo que está acelerando tiene masa:

Hemos observado que la masa, tal como la observa un observador externo, varía en razón de

1 1 v 2 / C 2
Entonces, nuestra observación dice que la masa tiende a infinito cuando v se acerca a c , pero no sabemos "por qué", exactamente, pero actúa como una resistencia creciente para viajar a través de un "medio" o rendimientos decrecientes tratando de agregar impulso a la objeto de viaje. En función de la energía requerida, aumenta la velocidad, su rendimiento en el aumento de la velocidad es cada vez menor porque la resistencia al movimiento, "masa", se hace cada vez más grande y, finalmente, debido a la "masa", la energía (que son ondas o materia). en el mismo medio que el objeto que estamos acelerando, por lo que debe viajar más lento que la velocidad de la luz) eventualmente se vuelve tan ineficaz que realmente no causa un efecto para ninguna fuente de energía conocida.

2) Lo que está acelerando no tiene masa:

Lo único que conocemos actualmente que no tiene masa, pero que se puede medir de alguna manera, es una cosa de onda/partícula, y no puede acelerar, simplemente viaja a través del fluido del espacio-tiempo. La gravedad, EM como la luz, las fuerzas fundamentales se propagan como una onda en un medio. Y de nuevo, ese medio es "Espacio-tiempo" pero no sabemos de qué está hecho. Otras ondas viajan a través de la materia y hay una velocidad de onda que se puede medir en todas ellas. PERO, es un viaje en la materia, dentro del espacio-tiempo.

3) El tercer caso de aceleración de la materia más allá de la velocidad de la luz es una especulación interesante, pero en última instancia inútil, ya que no es posible según la física conocida, ya que no hay forma de agregar más impulso después de llegar a la velocidad de la luz. Si alguien agregara una fuerza que empujara algo que no sea "espacio...", hay un experimento mental para un viaje más rápido que las ondas en el medio del espacio-tiempo.

Si observa el viaje más rápido que la estela en otro medio, puede ver que es posible, pero las cosas cambian. Por un tiempo, pensamos que no podíamos viajar más rápido que el sonido, pero debido a que no hay un límite de aceleración, sucede algo especial en el límite, un avión puede sobrepasarlo y provoca una onda de choque cónica detrás de él. Un bote puede viajar más rápido que las olas en el agua, y nuevamente, una ola rompiendo, una "onda de choque" aparece en la superficie detrás del bote. ¿Qué significaría acelerar para viajar más rápido que las ondas que viajan en el espacio-tiempo? No creemos que se pueda hacer, pero parece que sabemos cómo se vería a partir de un viaje relativista FTW de menor velocidad. Hay un nombre para la onda de choque creada cuando la materia viaja más rápido que la velocidad local de la luz (un reactor de fisión es el ejemplo habitual), llamada radiación de Cherenkov, que rastrea con normal más rápido que el viaje de onda en un fluido. (Vemos luz visible, un bonito tono de azul). Para viajar en fluidos normales, viajar más rápido que una ola provoca un nuevo régimen de estela. Con la velocidad de la luz, la velocidad de una onda en el espacio-tiempo, ¿podría ser lo mismo? Tal vez, si hubiera alguna otra fuerza, alguna otra forma de agregar impulso al sistema. Algo que viaje más rápido que c podría potencialmente formar una onda de choque y tal vez deformar el espacio-tiempo de una nueva manera. Hay indicios de que el espacio-tiempo actúa como un fluido comprimible por masa; Si excluyó la gravedad como una fuerza que podría actuar directamente sobre la luz, la luz que se curva alrededor de una masa lo suficientemente grande parece lo que sucede cuando la velocidad de un objeto cambia en la dirección perpendicular a su viaje, lo que llamamos refracción. Probablemente sea una comparación equivocada, sin embargo, dado que no hay evidencia de que la luz PUEDE acelerar; de lo contrario, el horizonte de sucesos alrededor de un agujero negro no se tragaría la luz. Pero hay gente mirandodensidad del espacio Entonces, hay investigación alrededor de los bordes, pero rompiendo ese límite de c No es tan probable, pero es interesante considerarlo.

No estoy de acuerdo con el punto de que existe un límite de velocidad universal en nuestro universo, a menos, por supuesto, que defina la palabra "velocidad" de una manera muy poco intuitiva.

Afirmo que dividir la longitud adecuada de la distancia recorrida en el marco del observador por el tiempo adecuado en el marco del objeto ( = γ v ) está más cerca de nuestra comprensión intuitiva de la velocidad que la variable v (que se define con respecto a la definición poco intuitiva de tiempo en la relatividad especial). Esta cantidad no está limitada por C .

De manera equivalente, uno puede tomar la magnitud del vector que consta solo de los componentes espaciales de las cuatro velocidades. De nuevo, esto no es igual a v y puede ser mas grande que C .

Estoy confundido: la magnitud al cuadrado de la parte espacial del vector de velocidad 4 es γ 2 v 2 = v 2 / ( 1 v 2 / C 2 ) . Si v > C como sugiere, esta cantidad se vuelve negativa, lo que no tiene sentido para la magnitud al cuadrado de este vector.
¿Qué pruebas tienes de que d v / d τ es ilimitado?
@ZeroTheHero No es v lo que se vuelve más grande que c sino gamma veces v.
(Lo cual puede verificar fácilmente por sí mismo @KyleKanos).
@MrFrety: No estoy seguro de lo que quiere decir, ¿cómo puedo verificar esto yo mismo?
@KyleKanos Espera un segundo, estás hablando de una aceleración. Supongo que quisiste escribir x en lugar de v. Para elaborar un poco más: ZeroTheHero anotó correctamente el cuadrado de la cantidad de la que estoy hablando. Intente, como hice yo, poner algunos números muy cerca de 1 para v en γ v mientras establece c en 1. Encontrará que esta expresión puede ir arbitrariamente mucho más allá de 1 (= c).
@MrFrety: Ups, sí. yo había querido decir X . Perdón por la confusión :(
+2 Sí, esta es una idea obvia cuando comienzas a reflexionar sobre la relatividad. Lástima que este sitio sea intolerante con el pensamiento no convencional, por lo que la mayoría de las respuestas creativas reciben votos negativos. Sigan con el buen combate. Ahora tienes la reputación de comentar en cualquier lugar.

La teoría del taquión podría aportar algunas ideas a esta pregunta. De acuerdo con la teoría de los taquiones, es posible que una partícula pueda viajar más rápido que la velocidad de la luz siempre que la partícula se haya creado con una velocidad mayor que la velocidad de la luz. Y el límite de la velocidad de la luz, según la teoría de los taquiones, puede acercarse desde abajo o desde arriba, pero no cruzarse. Un taquión con velocidad infinita tendrá energía relativista cero , pero ganará energía a medida que desacelera hacia el límite de la velocidad de la luz.

Pero una velocidad tan rápida como la de la luz tiene propiedades temporales extrañas, las partículas viajan tan rápido que constantemente retroceden en el tiempo, y siempre vendrán del futuro e irán al pasado. Visto desde nuestra realidad, esto podría verse como partículas que viajan constantemente a través de agujeros de gusano hacia el pasado, y nunca hemos observado tales partículas, y es posible que ni siquiera sea posible observar tales partículas.

Mediante el uso de láseres y el entrelazamiento cuántico, se ha medido que el enlace cuántico es al menos 10 000 veces más rápido que la luz , por lo que tiene una velocidad infinita o casi infinita. A Einstein no le gustaba el entrelazamiento cuántico, ya que parece romper su límite de velocidad de la luz. No sabemos qué es exactamente el vínculo cuántico, algunos sugieren agujeros de gusano, algunos sugieren superposición, algunos dicen que está vinculado en un plano fundamental, ondas libres de partículas, etc., pero si el vínculo cuántico involucra partículas viajeras, estas partículas deben ser taquiones. Partículas con velocidad infinita o casi infinita.

Y es extraño que en la mecánica cuántica el observador pueda influir en el resultado de un experimento. El resultado del experimento sucedió en el tiempo, ya que toma unos nanosegundos antes de que la luz nos alcance para que podamos observar el resultado del experimento. Dado que la observación influye en un resultado, ¿es la observación como elegir potenciales cuánticos pasados ​​del futuro? Y hablando relativamente, ¿realmente el experimento está ocurriendo en el pasado? Estas cosas a menudo me confunden.

Richard Feynman y Wheeler tenían una hipótesis extraña, ¿qué pasa si la luz es emitida por un fotón avanzado que viajó en el tiempo y emite la luz? Podríamos argumentar que esto no es romper la causalidad ya que para esta partícula el tiempo lineal va en sentido contrario. Esta respuesta de Terry Bollinger explica maravillosamente esta hipótesis: ¿ Se puede emitir un fotón sin un receptor?

Inspirado por Feynman, me pregunto si hay una relación aquí, si se emite un taquión con una velocidad casi infinita y una energía cercana a cero, desde nuestro ojo en el mismo momento en que se recibe una luz, ¿podría el taquión viajar atrás en el tiempo hasta el emisor e influir en el salto cuántico que emitió la luz? La relación es entonces: ¿La velocidad infinita coincide con la velocidad de la luz en dirección inversa? Alguien sabe como calcular esto?

Si existe tal relación, entonces podríamos preguntarnos si la velocidad infinita juega un papel en el establecimiento del límite de la velocidad de la luz.